NARM Study Questions

Pataasin ang iyong marka sa homework at exams ngayon gamit ang Quizwiz!

pulmonary artery

----------- brings deoxygenated blood to the lungs from the heart - pulmonary vein - pulmonary artery - alveoli - bronchiole

mild late decels

15bpm drop from baseline to nadir

moderate late decels

16-45 bpm drop from baseline to nadir

chest circumference of newborn

2 cm or .75 inches less than head (13.5-14 inch)

4th Degree laceration of perineum

-through rectal sphincter

2nd Degree laceration of perineum

-vaginal mucosa and initial perineal muscle

in maintains the fetus's temperature

A first time mother asks the midwife, "What does the bag of waters do during pregnancy?" The midwife's best response is: Answer "It helps the fetus obtain nourishment from the placenta" "It protects the fetus's skin" "It maintains the fetus's temperature" "It helps to develop your baby's eyes"

Rubella (German Measles)

A highly contagious viral disease, especially affecting children, that causes swelling of the lymph glands and a reddish pink rash; may be harmful to the unborn baby of a pregnant woman who contracts it

-active sexually transmitted infection

A risk factor that may appear as a pre-existing medical disorder would be: -history of hemorrhage -active sexually transmitted infection -high stress living conditions -history of abortion

non-immunity

A rubella titer of <1:10 indicates: Answer immunity non-immunity need for an immunization infection

infection

A rubella titer of >1:64 indicates: Answer -infection -non-immunity -need for RhoGam -need for immunization

A woman pregnant for the first time

A rubella titer should be done on: -a woman pregnant for the first time - a woman who has chicken pox in the final month of her pregnancy - a woman who can prove she was immunized against german measles

-the baby has irregular/difficult breathing

A score of 1 is given on Apgars for respirations if: Answer -the baby is not breathing -the baby has irregular/difficult breathing -the baby is crying -the baby is cyanotic

Leopold's Maneuvers

A series of four maneuvers designed to provide a systematic approach whereby the examiner may determine fetal presentation and position.

mismanagement of the third stage

All of the following are predisposing factors that may lead to a hemorrhage, except: Answer -mismanagement of third stage -grand multiparity -multiple pregnancy -history of postpartum hemorrhage

alpha fetoprotein

All of the following birth defects could be detected by genetic screening, except: Answer -autosomal abnormalities -neural tube defects -thalassemia -alpha fetoprotein

-a pre-existing condition of high blood pressure

Chronic hypertension refers to: Answer -a pre-existing condition of high blood pressure -high blood pressure brought on by pregnancy -a psychological condition occurring in the postpartum stage -high blood pressure brought on by the medical examiner

Mechanisms of Labor

Descent Engagement Flexion Internal rotation Extension External rotation Expulsion

DIC (disseminated intravascular coagulation)

Condition affecting the blood's ability to clot and stop bleeding.

--velamentous

Description of the placenta when the blood vessels in the umbilical cord separate and leave the cord prior to insertion into the surface of the placenta: Answer battledore vasa previa succenturiate velamentous

fimbriae

Fingerlike projection of the uterin (fallopian) tubes that drape over the ovary. - plicae - fimbriae - rugae - chordae

-the destruction of all forms of microbial life

For purposes of microbial control, the term sterilization implies: -the removal of pathogenic forms of microorganisms -the lowering of microbial count -the destruction of all forms of microbial life -the destruction of microorganisms only on the body surface

a woman with a previous tubal pregnancy

For which of the following pregnant women is watchful waiting not appropriate when there is painless vaginal bleeding in the first trimester? Answer -a woman with two previous spontaneous abortions -a woman with previous tubal surgery -a woman who is pregnant with twins -a woman with a suspected blighted ovum

-the beginning of one contraction to the beginning of the next

Frequency of contractions is timed from: Answer -the end of one contraction to the end of the next -the beginning of one contraction to the end of the next -the end of one contraction to the beginning of the next -the beginning of one contraction to the beginning of the next

2nd stage of labor

Full dilation --> birth of baby

-prepare to suction only if the baby is not vigorous at birth

Ginger's membranes rupture during the second stage of labor, revealing moderately meconium stained amniotic fluid. The midwife will prepare to: Answer -transport to the local hospital for fetal distress -prepare to suction with a bulb syringe rather than a delee mucus trap -prepare to suction with a delee mucus trap on the perineum -prepare to suction only if the baby is not vigorous at birth

--hypothermia

In newborns, an early sign of sepsis is: Answer hypothermia hyperglycemia jitteriness crying

40-45%

In pregnancy, blood volume increases by approximately: Answer 40-45% 10-15% 0% 79-80%

----estrogen

In response to anterior pituitary stimulation, the graafian follicle secretes: Answer progesterone estrogen prolactin glycogen

rises for a short time then falls

In shock, a person's blood pressure typically: Answer -rises for a short time then falls -falls but then rises again -stays the same -rises to 140/90 and stays there

rises

In shock, a person's pulse: Answer rises falls stays the same falls to 50 and stays there

rise

In shock, a person's respirations: Answer rise fall stay the same become strong and fast

-placenta previa

In the second or third trimesters, vaginal bleeding most often would indicate: Answer -ectopic pregnancy -placenta previa -spontaneous abortion -implantation bleeding

-frank

In this type of breech, the bottom comes first, with the legs flexed at the hip and extended at the knees (with feet near the ears): complete frank footling kneeling

-they may have cognitive disabilities

Low birth weight is defined as a baby born at term weighing less than 5 1/2 pounds, or 2500 grams. Which of the following is true about low birth weight babies? Answer -they always have immature lungs -they have healthier hearts because they do not have too much fat -they may have cognitive disabilities -they may become hyperglycemic

amniotic fluid embolism

Robin is a G1P0 who has been in labor for 20 hours. Her water broke 5 hours ago. She suddenly complains of chest pain and has pink, frothy sputum and diaphoresis. These may be early signs of: Answer -HELLP syndrome -amniotic fluid embolism -disseminated intravascular coagulation (DIC) -pneumonia

-the fetus may have experienced oxygen deprivation at some point, or is post mature

The membranes break during labor and the amniotic fluid is reported on the chart as "brown tinged." This indicates that: Answer -the fetus has an infection -the fetus may have experienced oxygen deprivation at some point, or is post mature -the fetus is in distress and should be delivered immediately -the fetus is most likely in the breech position and the midwife should prepare for resuscitation

-the baby is under 6 months old

The lactation amenorrhea method of birth control is typically reliable as long as: Answer -the baby is under 6 months old -the woman is breastfeeding twice a day -the woman has her periods -the woman has other children

myometrium

The largest portion of the uterus is the: Answer myometrium endometrium perimetrium decidua

-is flat on her back with her legs in stirrups

The lithotomy position is when the mother: -is flat on her back -is left side lying -is flat on her back with her legs in stirrups -is semi-sitting

--uterine atony

The major cause of postpartum hemorrhage is: Answer infection uterine atony cervical or deep vaginal tears malnutrition

maintain the corpus luteum in the first trimester

The primary function of human chorionic gonadotropin is to: Answer -confirm the diagnosis of pregnancy -maintain the corpus luteum during the first trimester -nourish the fetus -stabilize the uterus during pregnancy

>90 degrees

The pubic arch of the platypelloid pelvis is: Answer >90 degrees <90 degrees = 90 degrees a narrow arch

-determine what is in the pelvis

The purpose of Leopold's maneuvers is to: -determine what is in the pelvis -record the degree of descent -estimate the fetal weight -determine breech scoring at term

-determine fetal lie and presentation

The purpose of Leopold's maneuvers is to: Answer -feel the mother's stomach tone -check her for linea negra -estimate fetal weight -determine fetal lie and presentation

-attitude

The relation of the fetal head to its trunk is: Answer flexion attitude presentation posture

position

The relation of the occiput (in a vertex presentation) to the 8 areas of the pelvic brim is called the fetal: Answer presentation lie position cephalic prominence

to help direct the mother in the most responsible choice

What is the ultimate aim of prenatal counseling? Answer -to help direct the mother in the most responsible choice -to elicit cooperation -to engage midwife and client trust -to be open and share beliefs and ideas about birth and mothering

50%

What percentage of ABO incompatibility is linked to mothers with O type blood? -30% -40% -50% -60%

7am

What time of day are HCG concentrations highest?

Microcytic anemia

What type of anemia displays these values? small and pale RBC Low MCV and MCHC, low hemoglobin, low ferritin - Megaloblastic - Microcytic - normocytic

megaloblastic anemia

What type of anemia is commonly associated with the following values large nucleated RBC, Increased MCV, MCH, MCHC, serum folate <3ng/mL - Megaloblastic -microcytic -normocytic

-some fetal or placental parts are passed, but some remain inside

When an incomplete miscarriage occurs: Answer -there is light spotting and cramping and the cervix remains closed -the baby has died in the uterus, but the cervix remains closed -some fetal or placental parts are passed, but some remain inside -the fetus is reabsorbed

location

When assessing labor contractions, the least important for the midwife to describe is: Answer location intensity duration frequency

-"Your burping technique is good, but try burping him only at the end of the feeding"

When breastfeeding her newborn son, Talitha, a first time mother, attempts to burp him after every few swallows of milk. He begins to cry and Talitha becomes upset and frustrated. Which response by the midwife is most appropriate: Answer -"You're burping him much too frequently" -"Just look how upset your baby has become. Give him to me, I will calm him down" -"Your burping technique is good, but try burping him only at the end of the feeding" -"He's confused; try putting him on your shoulder to burp him"

SOAP

When charting, which of the following are acceptable? Answer writing entries with pencil white-out to correct mistakes minimal details SOAP notes

vitamin C

Which nutrient maintains collagen, helps heal wounds and fractures, strengthens your blood vessels, aids with iron absorption, and reduces lipid production in the brain? - Vitamin A - Vitamin c - folic Acid - Vitamin K

fatigue

Which of the common discomforts of pregnancy generally have their initial onset during the first trimester of pregnancy? -backache -dyspnea -fatigue -varicose veins

freely indulge in favorite foods

Which of the following are not recommended as a component of self-care in pregnancy? Answer -freely indulge in favorite foods -drink at least two quarts of water every day -vent any frustrations before they become explosive -do exercises specific to pregnancy several times a week

fetal heart tones heard with a doppler

Which of the following is considered a positive sign of pregnancy? Answer -movement -auscultation of fetal parts -fetal heart tones heard with a doppler -increase in maternal weight gain

iron

Which of the following is essential in building blood volume levels of hemoglobin and assists in metabolism of protein? Answer vitamin K protein iron zinc

elimination of the need for the vitamin k injection

Which of the following is not a benefit of initiating breastfeeding soon after delivery: Answer -decreased risk of infant hyperbilirubinemia -enhanced maternal infant bonding -enhanced uterine involution -elimination of the need for vitamin K injection

zero station

While performing a vaginal exam, the midwife determines that the fetus is in a cephalic presentation and that the occiput has reached the level of the ischial spines. The station is: -2 (minus two station) -1 (minus one station) 0 (zero station) +1 (plus one station)

feel cool and clammy

Women in shock after blood loss following childbirth will often: Answer -want to sit up and eat -feel hot and feverish -feel a surge of energy right before passing out -feel cool and clammy

-abruptio placenta

Women with a diagnosis of severe preeclampsia have an increased risk of: -complete abortion -placenta previa -abruptio placenta -retained placenta

Type O

You are typing baby jack's blood. you place two samples on slides and add anti-A plasma to one and anti-B plasma to the other. Neither sample agglutinates. What is baby Jack's blood type? - Type A - Type B - Type AB - Type O

oophorectomy

Your mother is having her uterus surgically removed along with her ovaries. Removal of ovaries is called - hysterectomy - orchidectomy - oophorectomy - gasterctomy

A High false positive rate

Zelda, a G2P1 just came in to the office crying. She got a call from her doctor's office saying that her maternal serum alpha-fetoprotein screening test came back positive. The midwife can tell her it is important to remember that this test has: Answer a high false-positive rate low sensitivity a high false-negative rate high specificity

presumptive sign

_____ are those the woman notes as changes in her body which are commonly but not exclusively associated with pregnancy - positive - probable - presumptive

probable sign

_____ are those which are usually due to pregnancy but may also be present in other conditions - positive - probable - presumptive

artery

_______ is a muscular blood vessel that carries blood away from the heart - vein -artery - capillary - plasma

antibody

___________ are proteins produced in the body in response to the presence of antigens or foreign bodies - plasma - antigens -arterioles -antibody

anecephalic

a baby is born without a brain. The diagnosis would be: -anecephalic -encephalitis -meningocele -meningomyelocele

hydatidiform mole

a benign tumor of the placenta consisting of multiple cystic and resembling a bunch of grapes

ductus arteriosus

a blood vessel in a fetus that bypasses pulmonary circulation by connecting the pulmonary artery directly to the ascending aorta

circumoral cyanosis

a bluish discoloration around the mouth

cyanosis

a bluish discoloration of the skin resulting from poor circulation or inadequate oxygenation of the blood.

a person with a non expressed gene for a trait which can be passed to offspring

a carrier is - a person with a variation in the number of chromosome pairs - a person with a non expressed gene for which a trait can be passed to offspring - the pattern of inheritance passed through families

symphysis pubis

a cartilaginous joint that is the point of fusion for two pubic bones

adheres to the cervix by suction

a cervical cap: - adheres to the cervix by suction -stays on only when spermacide is applied -should be inserted 6 hours prior to intercourse -rests in the posterior fornix and against the back of the symphisis pubis

-trisomy 21

a child born to a woman over the age of 35 has an increased risk of - trisomy 21 -cleft palate - cystic fibrosis

hypotonia

a condition in which there is diminished tone of the skeletal muscles

oliguria

a condition of decreased urine - oliguria -proteinuria - bacteremia - dsypnea

Down Syndrome

a condition of intellectual disability and associated physical disorders caused by an extra copy of chromosome 21.

supine hypotension

a drop in blood pressure due to altered venous return from a gravid uterus exerting pressure on the ascending vena cava

20 weeks

a fetus weighing about 299 grams and measuring 17.5 cm in length that can actively suck and swallow amniotic fluid is approximately how many weeks gestation? - 14 weeks -16 weeks - 20 weeks -28 weeks

hematuria

a lab report comes back to you documenting numerous erythrocytes in the urine specimen. the term for this is. - hematuria -anuria -oliguria -nocturia

essure

a method of tubal sterilization that blocks the oviducts by use of metal coils

breast infection

a mother who gave birth two weeks ago calls to report that she was a 103 fever, chills, a headache and body aches what is most likely the cause of her symptoms? - laceration infection -breast infection - respiratory infection - uterine infection

dehydration or exertion associated with labor

a normal temp rise may occur up to 2 hours after birth as a result of - dehydration or exertion associated with labor - breast engorgement - shock - weight loss

HIV

a positive ELISA test signifies the possible presence of what: - HPV - Syphilis - e-coli - HIV

administer magnesium sulfate

a pregnant woman at 38 weeks has pitting edema. THe midwife should do all of the following except - evaluate her reflexes, her BP, and her urine for protein - keep her on her left side during rest periods - administer magnesium sulfate - monitor vital signs more frequently

grand mulitpara

a prenatal factor that may cause hemorrhage is: - grand multiparity -mismanagement of the third stage - uterine atony -baby not able to nurse right away

utero-placental insufficiency

any condition in which the placenta is not able to supply sufficient oxygen and nutrients to the fetus - can be chronic or acute - associated with postmaturity

early decelerations

appear in active labor and are associated with moderate variability they are benign and do not require intervention - decels that occur at the peak of a contraction

APGAR score components

appearance, pulse, grimace, activity, respiration

milk

approx 1/3 of riboflavin is supplied in the american diet by? - meat - milk - fruit -rhubarb

amniotomy

artificial rupture of membranes

depositing layers of subcutaneous fat

at 37 weeks a baby is most likely: - finished developing cells in the brain - developing hair and fingernails - thickening skin layers for thermal insulation - depositing layers of subcutaneous fat

both b and c

controlled cord traction to deliver the placenta should only be commenced when: a. the uterus is not contracted b. the woman is not in an upright position c. the uterus is brace d. both a and c e. both b and c

cranial sutures

coronal, sagittal, lambdoid, squamous

a decrease in peripheral resistance caused by the influence of progesterone

in pregnancy, blood volume increases but blood pressure usually decreases. What causes this result? - a decrease in peripheral resistance caused by the influence of progesterone - a decrease in cardiac output - an increase in peripheral resistance caused by the influence of progesterone - a decrease in peripheral resistance caused by the influence of relaxin

Diaphragm (birth control)

dome-shaped device inserted into vagina and placed against cervix prior to sex. the sponge contains a spermicide. must stay in place 6 hours post coitus

B VITAMINS

due to inhibition of transport mechanisms by progesterone, the absorption of which of the following nutrients is decreased in the small intestines during pregnancy - iron - glucose - water - b vitamins

Proliferative phase

during one phase of the uterine cycle the functional endometrium rebuilds, preparing for a possible pregnancy. What is the name of this phase? - proliferative phase - luteal phase - secretory phase - menstrual phase

nausea and vomiting caused by an increase in smooth muscle tone of the upper GI tract

during pregnancy there are several physiological changes of the digestive system that take place. Which of the following statements is incorrect? -nausea and vomiting caused by an increase in smooth muscle tone of the upper GI tract - heartburn due to relaxation of the esophageal sphincter and increased pressure gradient - increase absorption time of some nutrients caused by decreased motility of the small intestines - constipation caused by decreased motility of the large intestines leading to increased water absorption

secretes progesterone to maintain the pregnancy

during the first 10-12 weeks of pregnancy, the corpus luteum - gradually progresses and become obliterated - secretes estrogen to maintain the pregnancy - secretes hCG to maintain the pregnancy -secretes progesterone to maintain the pregnancy

line up along the equatorial plane

during the metaphase of mitosis, chromosomes - line up along the equatorial plane - change into chromatin - migrate to opposite poles of the cell - are surrounded by a nuclear membrane

exerting downward pressure on the top of her fundus while guarding her uterus above the pubic bone

immediately following the birth of the placenta, the midwife goes to rub the fundus to maintain it in a contracted state: she does this by - pushing down in the cetner of the uterus in an up and down motion - massaging her stomach lightly in the epigastric area - exerting downward pressure on the top of her fundus while guarding her uterus above the pubic bone - using one hand to rub in a counter clockwise motion and then reverse to a clockwise motion

7-9

implantation occurs about how many days after fertilization - 1 - 2-3 -4-5 - 7-9

6-7

implantation usually begins on which day following fertilization - 1-2 -3-4 - 4-5 - 6-7

zona pellucida/ corona radiata

at ovulation the oocyte is surrounded by 2 layers in which the deepest layer is the ___________ and the out layer is the_____________ - zona pellucida/corona radiata - corona radiate/ zona pellucida - zona pellucida/ acrosome -acrosome/ corona radiata

composed of 15-20 cotyledons and approx 1/5- 1/6 of the baby's weight

at term the placenta is - 2-4 inches in diameter and 1 inch thick at the center - 6-8 inches in diameter and composed of 5-10 cotyledons - composed of 10-15 cotyledons and 8-10 inches in diamter - composed of 15-20 cotyledons and approx 1/5 - 1/6 the weight of the baby

16 weeks

at what week of gestation do genetic and environmental factors begin to have an effect on fetal growth - 8 weeks - 12 weeks - 16 weeks - 20 weeks

26-29 weeks

at which weeks do the following fetal developments occur: eyes open, CNS can control breathing, lungs capable of breathing air - 13-16 - 9-12 - 26-29 - 21-25

35-38 weeks

at which weeks do the following fetal developments occur: firm grasp, circumference of head and abdomen are usually equal - 16-20 weeks - 26-29 weeks - 35-38 weeks - 9-12 weeks

17-20

at which weeks do the following fetal developments occur: lanugo covers the skin, skeletal muscles become active so mother may begin to feel movement, vernix appears on skin

9-12 weeks

at which weeks do the following fetal developments occur: urine formation begins, ossification centers appear in most bones, eyes fuse, body growth is accelerated, fetus begins swallowing amniotic fluid - 13-16 weeks - 9-12 weeks - 20-25 weeks - 36-38 weeks

21-25 weeks

at which weeks do the following fetal developments occur: fingernails present, rapid eye movement begins, startle response to noise, surfactant production begins - 12-20 weeks -21-25 weeks - 26-30 weeks - 35-40 weeks

dominant

autosomal traits that will be expressed if at least 1 gene is present for that trait is a ______ - recessive - dominant -phenotype - allele

tongue

glossitis defines inflammation of the -mouth -vagina -gums -tongue

glucosuria

glucose in the urine

Friedman's Curve

graph used to plot rates of cervical dilation and fetal descent during active labor; should dilate approx. 1cm/hr, if not then further monitoring/ intervention may be required (it's stupid and inaccurate)

vitamin k- food sources

green leafy vegetables, egg yolks, safflower oil, molasses, cauliflower, soybeans

calcium-food sources

green leafy vegetables, shellfish, molasses, sesame seeds, seaweeds

vertex

head down baby

4 sperm cells with 23 chromosomes each

in spermatogenesis, meiosis results in the formation of - 1 sperm cell with 23 chromosomes - 2 sperm cells with 46 chromosomes each - four sperm cells with 23 chromosomes each - four sperm cells with 46 chromosomes each

estrogen and progesterone are low

in the female reproductive cycle , menstrual flow occurs when the concentration of______ - estrogen and progesterone are low - estrogen and progesterone are high - estrogen is high and progesterone is low - progesterone is high and estrogen is low

the corpus luteum

in the female reproductive system which of the following secretes progesterone - the pituitary gland - the corpus luteum - the graafian follicles -the fallopian tubes

retraction and contraction

in what ways do the uterine muscles act to dilate the cervix -detraction and contraction -retraction and contraction -contraction and subinvolution -flexion and descent

secondary infertility

inability to conceive after a previous pregnancy

primary infertility

inability to conceive after one year of engaging in unprotected sex

diuresis

increased formation and secretion of urine

hypertonus

increased tension, as muscular tension in spasm

pelvic outlet

inferior opening of the true pelvis

pelvic inflammatory disease (PID)

inflammation and infection of organs in the pelvic region; salpingitis, oophoritis, endometritis, endocervicitis

thrombophelbitis

inflammation of a vein associated with a blood clot

phlebitis

inflammation of a vein is called - lymphangitis - phlebitis - arteritis - angina

cystitis

inflammation of the bladder is called - cyanosis - cystitis -cystocele - rectocele

mastitis

inflammation of the breast; most commonly occurs in women who are breastfeeding

endometritis

inflammation of the inner lining of the uterus

metabolic alkalosis

ingestion of excessive amounts of sodium bicarbonate to relieve symptoms of indigestion may lead to - metabolic acidosis - metabolic alkalosis - respiratory acidosis - respiratory alkalosis

groin

inguinal refers to which body part - armpit - groin - knee - ankle

plane of least dimensions

interspinous diameter -approx 10cm

connective tissue known as whartons jelly, plus high blood volume pulsing through the vessels

intrauterine compression of the umbilical cord is prevented by - a cushion of amniotic fluid plus fat stores in the cord - a thick layer called the mesodermal layer, plus high intrauterine blood pressure pulsing through the cord - connective tissue known as whartons jelly, plus high blood volume pulsing through the vessels - high absorbent vessel walls, plus two way circulation through the cord

vasospasm

involuntary contraction of a blood vessel

1st degree laceration

involves only skin and superficial structures above muscle

scalp color pinkish

jesse, a primigravida has been pushing for 2 hours. her baby is finally beginning to crown, but you are unable to get FHT. which of the following is a good indicator of fetal wellbeing at this stage? - scalp color pinkish - scalp color purpleish - scalp color blueish - fetus is very active

gestational hypertension

high blood pressure during pregnancy

frank breech

hips flexed, knees extended

face down within 5-10 seconds

how and how soon should the newborn be lifted from the water following a water birth? - immediately feet first to drain water from the mouth and nose - face down within 5-10 seconds - as soon as the cord stops pulsing - face down within 30 seconds

by fetal respiratory secretions and urine

how is amniotic fluid primarily formed by 20 weeks of gestation - by amnioblast cells - by fetal respiratory secretions and urine - from maternal tissue and fluid diffused across amniochorionic membranes from the decidua parietalis

2.5cm

how long is the average non pregnant cervix - 2.5cm - 1.5 cm - 3.5cm - 2 inches

1 in 10

how often do miscarriages occur after pregnancy has been clinically diagnosed - 1 in 2 -1 in 5 -1 in 10 -1 in 12

dry and stimulate

if a baby comes out floppy with labored breathing the first thing to o would e to initiate: -PPV - Free flow oxygen - dry and stimulate - chest compressions

diabetes

jessica is 24 weeks gestation. She is feeling great and rpts no problems. complete urinalysis shows the following. SG: 1.025, pH:6, - protein, +1 glucose, +1 ketones, - leukocytes, - nitrites, - blood what would you suspect? - reduced renal threshold for glucose - dehydration - diabetes - probable vaginal contamination

posterior fontanelle

junction between the two parietal bones and the occipital bone; smaller; triangular shaped

biparietal diameter

largest transverse diameter of the fetal head; measured between the parietal bones

between 2 and 6 weeks pp due to retained membranes

late pp hemorrhage is most likely to occur: - immediately pp due to the baby not nursing right away - in third stage due to a sticky placenta that does not completely separate - between 2 and 6 weeks pp due to retained membranes - between 6 and 12 weeks pp due to subinvolution

potassium- food sources

lean meat, whole grains, vegetables, dried fruit, legumes, sunflower seeds

vitamin a- food sources

liver, eggs, yellow fruits/veggies, dark green fruits/veggies, dairy products

ischial spine

located superior to the ischial tuberosity and projects medially into the pelvic cavity

pink/brown in color

lochia serosa is present about 3-10 days following birth . it is best described as - bright red and profuse - pink/brown in color - dark red with some clots - yellowish and creamy

Sequelae

long-term or permanent damage to tissues or organs

production of sperm

male secondary sex characteristics include all of the following except: - thickening of vocal cords - increased muscular growth - production of sperm - increased growth of body hair

chronic hypertension

persistent high blood pressure, over 140/90 mm Hg

anaphase

spindle fibers transport sister chromatids towards the opposite poles of the cells during which phase? - prophase - metaphase -anaphase - telephase

neurological maturity

square window, politeal angle, and scarf sign are all test that refer to - physical maturity - the newborn exam - neurological maturity - psycholoigcal maturity

hot flash

sudden, temporary feeling of warmth, flushing, and sweating, often associated with menopause

pelvic inlet

superior opening of true pelvis; bordered by pelvic brim

salpingorrhaphy

surgical suturing of a uterine tube - salpingorrhaphy -salpingocele -salpingectomy -salpingography

brown fat metabolism

take place in fat cells themselves which are densely packed with mitochondria (brown color) and richly vascularized. So blood is warmed and transmits heat. 10X more efficient than shivering

apnea

temporary cessation of breathing

cause congenital malformations

teratogens are substances that - stimulate embryonic growth - cause congenital malformations - protect against infection - prevent accumulation of cholesterol

vulva

term for the collective external female genitalia - vulva -fourchette -Fornix - Labia Majora

AFP (alpha-fetoprotein)

test measured at 16-18 weeks gestation if serum increased indicates neural tube defects.

trisomy

the addition of a single chromosome is called - euploidy -aneuploidy - triploidy - trisomy

gestational age

the age of the fetus between conception and birth

phenotype

the appearance of expression of a physical feature or health condition related to specific genes is referred to as what? - genotype -phenotype - wild-type - karyotype

descending aorta

the arterial system of the fetus's lower body receives oxygenated blood flowing from the - ascending aorta - descending aorta - superior vena cava - inferior vena cava

capacitation

the biochemical and functional changes in sperm after entry into the uterus and uterine tubes is referred to as - decidualization - decondensation - capacitation - compaction

inner cell mass

the cells of a blastocyst that give rise to the body of the developing offspring constitute the - trophoblast -blastomere - inner cell mass - zona pellucida

a thick membrane that develops from the trophoblast

the chorion is: - a thick membrane that develops form the trophoblast - a thin protective membrane containing amniotic fluid - the endometrium after implantation - the second cavity of the blastocyst

autosomes

the chromosomes found in human somatic cells which do not carry gender determining genes are called what? - autosomes - sex-chromosomes - chromatids - alleles

apoptosis

the death of a cell as a result of natural causes is called - apoptosis - osmosis - anastomosis - erythroblastosis

s phase

the duplication of genetic material occurs during which phase of interphase - g1 phase -g2 phase - s phase - m phase

4-5 days before ovulation

the fertile period is considered to begin - 4-5 days after ovulation - 4-5 days before ovulation - 5 days before and after ovulation - on the day of ovulation only

Flexion

the fetal chin tips to chest

cardiovascular system

the first major system to function in the embryo is - the cardiovascular system - nervous system - reproductive system - respiratory system

menarche

the first menstrual period

zygote

the first mitotic division of the fertilized oocyte results in a - 2nd polar body - zygote - blastocyst - morula

quickening

the first movement of the fetus in the uterus that can be felt by the mother

the changing of the lochia beginning with alba, followed by serosa and finally rubra

the following are part of the continuing evaluation of the postpartum woman except: -vital signs, behavioral changes, responses to child bearing - s/s of obstetrical or medical complications - the changing of the lochia beginning with alba, followed by serosa and finally rubra

binding to an iron molecule in the heme part of hemoglobin

oxygen is transported in the blood by - inspiratory capacity increases - vital capacity increases - binding to an iron molecule in the heme part of hemoglobin - binding to proteins found within the plasma

Pulmonary arteries

oxygen poor blood is pumped from the heart to the lungs via which blood vessels - superior vena cava - inferior vena cava - pulmonary arteries - pulmonary veins

pelvic axis

path of baby during birth

plane of greatest dimension

pelvic inlet 13.5 cm

hematocrit

percentage of blood volume occupied by red blood cells

placenta percreta

perforation of uterus by placenta

releasing hormones from the hypothalamus

the secretion of homers from the anterior lobe of the pituitary gland is largely controlled by which? - releasing hormones from the hypothalamus - nerve impulses from the thalamus - nerve impulses from the hypothalamus - releasing factors from the thalamus

2nd lunar month

the sexual differentiation process does not occur in the fetus until the: -1st lunar month - 2nd lunar month - 3rd lunar month - 4th lunar month

epididymis

the site of sperm maturation is - seminiferous tubules - epididymis - ductus deferens - testis

metabolism

the sum of all processes in the body is known as - basal metabolic rate -metabolism -catabolism - anabolism

slightly moveable

the symphysis pubis and sacroiliac joints are described as? - slightly moveable - immovable - freely moveable - synovial

iodine

the thyroid is dependent on which of the following minerals in order to produce the necessary hormones - iodine - sodium - calcium - magnesium

muscle

the uterus is mostly composed of - muscle - fat - epithelium -connective tissue

high pressure gradient in veins, forcing blood towards the heart

there are several factors that contribute to the return flow of venous blood back to the heart. Which is not a correct factor? - constriction of venous walls during exercise - high pressure gradient in veins, forcing blood towards the heart - skeletal muscle contractions that squeeze the veins and open valves so blood can move towards the heart - venous valves that close to prevent blood from flowing away from heart

prolactin

this hormone does not play a part in the control of the ovary during the normal menstrual cycle but if produced in excessive amount such as during breastfeeding it will inhibit ovulation -oxytocin -estrogen -progesterone -prolactin

b vitamins

which nutrient is needed for carb, fat, and protein metabolism, aids your nervous system, keeps the muscle tone of the gastrointestinal tract, and ensures health of the hair, eyes, mouth, and liver? - carbs -b vitamin -fat - vitamin c

green and red peppers

which of the following are rich sources of vitamin C? - Milk and cheese - seafood - wheat and oats - green and red peppers

trisomy, autosomal, numerical disorder

which of the following best describes down syndrome - trisomy, sex-chromosome, structural disorder - monosomy, autosomal, numerical disorder - trisomy, autosomal, numerical disorder - monosomy, autosomal, structural disorder

secondary oocyte

which of the following can be fertilized to produce a zygote - secondary oocyte - primordial oocyte - primary oocyte - mature follicle

deliberately tightening a single muscle group as tight as possible and then letting it go as limp as possible and then repeating the exercise with different muscle groups

which of the following describes the technique of progressive relaxation - keeping one muscle group relaxed while another group is contracted - taking a deep breath and letting it out in a heavy sigh after a contraction - using mental imagery to distract thoughts from painful stimuli -deliberately tightening a single muscle group as tight as possible and then letting it go as limp as possible and then repeating the exercise with different muscle groups

formation of a 2 celled zygote

which of the following does not occur during and result in the completion of meiosis 2? - formation of a 2 celled zygote - increased oocyte metabolism - formation of the oocyte pronucleus - formation of the second polar body

ylang ylang

which of the following essential oils has been identified as helping to reduce BP in pregnancy at term? -ylang ylang -peppermint -rosemary -thyme

chickpeas, meat, fish

which of the following food groups contain the best sources of B6? - chickpeas, meat, fish - sweet potatoes, carrots, squash - spinach, swiss chard, mustard greens - raspberries, blackberries, cranberries

sardines, tofu, collard greens

which of the following food groups contains the best source of calcium - butter, cottage cheese - sardines, tofu, collard greens - eggs, sweet potatoes

Meat, fish, dairy

which of the following food groups provide the best sources of vitamin B12 - potatoes, rice, corn - beans, lentils, peas - strawberries, peppers, kale - meat, fish, dairy

strawberries, sweet red bell peppers, and brussel sprouts

which of the following foods contain the best sources of vitamin C? - strawberries, sweet red bell peppers, brussel sprouts - beans, peas, peanut butter - bananas, blueberries, watermelon - beef, lamb, cheese, and eggs

beef, clams, pumpkin seeds, lentils

which of the following groups of foods contains the best sources of iron - spinach, broccoli, peppers - potatoes, oatmeal, honey - beef, clams, pumpkin seeds, lentils - oranges, grapefruit, kiwi

Follicle stimulating hormone

which of the following hormones exerts its effect primarily upon the reproductive organs - FSH - Adrenocorticotropic hormone - prolactin releasing factor - thyrotropin

warm water and massage during labor

which of the following is a non-allopathic pain remedy - warm water and massage during labor - deep breathing and local anesthetic for crowning - tylenol and essential oils for headaches - deep relaxation and epidural during transition

-ultrasound

which of the following is a positive sign of pregnancy - positive pregnancy test - enlargement of the abdomen and fetal movement -amenorrhea -ultrasound

lochia serosa is pink and lasts 6-8 weeks

which of the following is false - lochia rubra is bright red and lasts 2-3 days -lochia serosa is pink and lasts 6-8 weeks -lochia alba is white and begins the 2nd or third week after birth -lochia that lasts past 4 weeks may mean the mother is overdoing activity

stimulates growth of breast tissue

which of the following is not a function of relaxin? - stimulates growth of breast tissue - inhibits contractility of uterine smooth muscle - relaxes pelvic ligaments and joints - suppresses secretion of oxytocin until birth

supplies nutrients to the oocyte

which of the following is not a function of the zona pellucida - after fertilization occurs, prevents further penetration by multiple sperm - prevents early implantation of the blastocyst within a uterine tube - acts as a sperm receptor - supplies nutrients to the oocyte

offensive odor

which of the following is not a normal characteristic of pregnancy discharge - low ph - thick consistency - offensive odor - white color

there are spontaneous respirations

Positive-pressure ventilation of the newborn should continue until which of the following conditions are met? - hr is above 80bpm - hr is above 100bpm - there are spontaneous respirations - there are spontaneous respirations and the babys color is pink

-the building blocks for growth and development; formation of hormones, enzymes, and antibodies; source of heat and energy

Protein is an essential component in the diet and provides which of the following? Answer -provides energy, serves as a carrier for fat-soluble vitamins, necessary for carbohydrate metabolism -the building blocks for growth and development; formation of hormones, enzymes, and antibodies; source of heat and energy -essential components of energy for the body's functions, assists in the digestion and assimilation of food -essential for the growth and repair of body tissues, and for health of eyes, promotes germ killing enzymes, destroys carcinogens

newborn transition

Pulse 160-180 100-120 bpm Respirations are irregular, 60-80 Crackles Apnea (<15 sec.) Temp. decreases Infant is alert, startles then sleeps for 60-100 min. Meconium is passed

woods maneuver

Pushing posterior shoulder around toward fetal back

72 hours after birth

Rh-negative mothers should be given RhoGam no later than: Answer 24 hours after birth 36 hours after birth 72 hours after birth 7 days after birth

two times

Risk of Respiratory Distress Syndrome (RDS) is slight whenever the concentration of lecithin in amniotic fluid is at least how many times greater than sphingomyelin? Answer three times four times two times ten times

ischial tuberosity

receives the weight of the body when sitting

cycle beads

red- first day menstrual cycle white- fertility days (avoid intercourse) brown- sex is okay/pregnancy unlikely

fetal attitude

relationship of fetal parts to one another

Fetal Lie

relationship of the long axis of the fetus to the long axis of the mother

Pulmonary Vascular Resistance

resistance to blood flow out of the right ventricle created by the pulmonary circulatory system

fundus

rounded upper portion of the uterus

vitamin D- Food sources

salmon, sardines, herring, egg yolks, organ meat, fortified dairy products

evening primrose oil and sex

sarah is a primigravida at 40 weeks with a bishops score of 3. which of the following would be the best to ripen her cervix? - chiro visit for the webster technique - castor oil, three two ounce doses - blue and black cohosh, one dropper every two hours - evening primrose oil and lots of sex

iodine- food sources

seafood, kelp, iodized salt

sodium- food sources

seafood, table salt, celery, processed foods, dairy, seaweed,

magnesium- food sources

seafood, whole grains, dark green vegetables, molasses, nuts

causes peristalsis of the urinary tract in the fetus

smoking during pregnancy may harm the fetus in all of the following ways except - causes 4000 chemicals including lead to get into the mothers bloodstream -causes narrowing of the blood vessels in the umbilical cord - causes prematurity and low birth weight -causes peristalsis of the urinary tract in the fetus

uterine souffle

soft, blowing sound made by the blood in the arteries of the pregnant uterus and synchronous with the maternal pulse

carbohydrates- food sources

whole grains, sweeteners, fruits, starchy vegetables

the first menstrual period

"Menarche" is defined as: -the first menstrual period -the last menstrual period -the moment fertilization occurs -the cessation of pregnancy

Philosophy of care

"Midwives believe that every woman has a right to a safe, satisfying birth in the setting of her own choice, with respect for human dignity, cultural sensitivity, and informed choice." The quote above is an example of a: - code of ethics - definition of midwifery practice -philosophy of care

mid pelvis

*occupies the space between the inlet and outlet *snug, curved space that the fetus must travel to reach the outside *as the fetus passes through this small area, its chest is compressed, causing lung fluid and mucus to be expelled *expulsion removes the space-occupying fluid so that air can enter the lungs with the newborn's first breath

newborn hypoglycemia

- different for preterm and term infants - Term (first 3 days)- blood glucose levels of less than 40mg/dL - preterm - less than 25mg/dl

platypelloid pelvis

- flat inlet - pubic arch is the widest of all pelvic types - rare and not very conducive to vaginal birth - makes rotation and descent of fetal head difficult - less than 3% of all women have this shape

causes of utero placental insufficiency

- maternal hypertensive disorders - diabetes -hyperthyroidism - autoimmune disorders such as lupus - abnormal placentation

3rd Degree laceration of perineum

-mucosa, down to rectal sphincter

term infant

37-42 weeks gestation

severe late decels

46+ drop from baseline to nadir

ominous

A fetal heart rate that has no beat-to-beat variations is considered: Answer normal ominous tachycardia bradycardia

deep transverse arrest

A condition of vaginal delivery that occurs when the fetal head fails to rotate despite steady descent. This is common in oversized babies and women that have a flat pelvis

face presentation

A fetus that is head down in the uterus and is completely extended is: Answer -occiput presentation -military presentation -brow presentation -face presentation

making decisions for the couple

A couple with a family history of an X-linked disorder ask for genetic counseling. The midwife is not responsible for: Answer -providing emotional support -making decisions for the couple -providing information about the disorder and possible results -serving as the couple's advocate

--dystocia

A difficult or obstructed labor: Answer dystocia eutocia dysuria dysmenorrhea

silent

A fetal heart rate that consistently stays the same with no variations is considered: Answer silent normal tachycardia bradycardia

uterine atony

A condition caused by failure of the uterine muscle to stay contracted after delivery: Answer uterine atony incompetent cervix subinvolution retained fragments

RDS

A condition in the newborn that is commonly associated with polyhydramnios is: Answer LGA RDS urinary problems FAS

-perineal skin

A 1st degree laceration involves the: Answer -perineal skin and muscle -perineal skin, muscle, and external anal sphincter -perineal skin, muscle, and rectum -perineal skin

halfway between the symphysis pubis and umbilicus

A 24 year old G1 P0 is in the midwife's office for her first prenatal exam at approximately 16 weeks gestational age. Where would the midwife expect to feel her uterine fundus? Answer barely above the symphysis pubis halfway between the symphysis pubis and umbilicus one to two fingerbreadths below the umbilicus at the umbilicus

the blood serum levels of maternal serum alpha protein, human chorionic gonadatropin, and unconjugated estriol, that can be used to screen for neural tube defects, Down's Syndrome, and other abnormalities

A MsAFP3, or Triple Screen test, gives the following information: - definitive results as to the presence of Down Syndrome - concise info based on the presence of specific hormonal markers -the blood serum levels of maternal serum alpha protein, human chorionic gonadatropin, and unconjugated estriol, that can be used to screen for neural tube defects, Down's Syndrome, and other abnormalities

Carcinogen

A cancer-causing substance

shoulder

A cephalic presentation includes all of the following, except: -brow -face -shoulder -vertex

gonorrhea

A chlamydia infection is often tested for at the same time as: Answer -syphilis -granuloma inguinale -gonorrhea -HIV

-suggest that she eat small, frequent meals

A client at 12 weeks gestation tells her midwife that morning sickness is lessening but she still feels nauseated in the afternoons. Which action would be most appropriate? Answer -suggest that she eat dry crackers before getting out of bed -suggest that she eat small, frequent meals -instruct her to drink skim milk with her meals and avoid beans -determine her food preferences

-to chart, in her own words, the reason she came in, the physical findings, assessment, and any recommendations by the midwife

A client comes in to the prenatal clinic at 32 weeks gestation reporting that she has not felt the baby move in 24 hours. Physical exam reveals FHT of 140, and all findings are within normal limits. The most appropriate way for the midwife to chart this visit would be: Answer -to describe objective findings and to not worry about describing her concern as it was unfounded -to chart, in her own words, the reason she came in, the physical findings, assessment, and any recommendations by the midwife -to document her worry in her own words, in order to decrease her concerns in the future -to chart, in her own words, the subjective findings and the midwife's care plan only as this did not end up being a fetal demise

-teaching her pelvic rock exercises

A client reports occasional lower back pain. The midwife rules out UTI and tells her it is a normal discomfort of pregnancy. Which measure would be most appropriate? Answer -teaching her spouse how to apply sacral pressure -encouraging her to take frequent long walks -teaching her pelvic rock exercises -suggesting that she sit in straight backed chairs

-hyperemia of the nasal mucous membranes

A client tells the midwife that she experiences intermittent nosebleeds. Which normal physiologic change of pregnancy is probably responsible? Answer -elevated hCG level -hyperemia of the nasal mucous membranes -vascular congestion of nasal turbinates from tissue edema -elevated blood pressure

assess FHT

A laboring woman's membranes rupture at the end of a contraction. The midwife's first action would be to: -assess FHT -change the bed so she will be comfortable -instruct the woman to push -perform a vaginal exam

a y-chromosome from the sperm and an x-chromosome from the ovum

A male child is the result of the union of: Answer :an x-chromosome from both the sperm and the ovum :an x-chromosome from the sperm and a y-chromosome from the ovum :a y-chromosome from the sperm and an x-chromosome from the ovum :a y-chromosome from both the sperm and ovum

amniotic fluid phospholipid levels

A midwife sends a postterm woman in her care for a biophysical profile. Which factor is not assessed by this test? Answer fetal breathing reactive fetal heart rate qualitative amniotic fluid volume amniotic fluid phospholipid level

above

A minus sign (-) next to the number representing the degree of descent indicates that the presenting part is where in relation to the ischial spines? Answer above below equal to under

invite her to talk in further detail about these concerns

A mother confides in her midwife that she is nervous about her partner's ability to support her in labor. Which of the following would be an appropriate response from the midwife? Answer -recommend she ask her partner not to attend the birth -tell her to hire a doula for additional support -invite her to talk in further detail about these concerns -assure her that no mother plans to rely on her partner for knowledgeable support, that is why she hired a midwife

-less than 18 months between pregnancies

A mother may be at risk for a complicated pregnancy and delivery if she has which of the following conditions: Answer -more than four years between pregnancies -less than 18 months between pregnancies -has had three previous children -has a singleton pregnancy

6

A newborn exhibits the following at 5 minutes: Heart rate of 110 bpmSlow, irregular breathing with grunting and retractionsSome flexion of the extremitiesA grimace in response to stimulusSome cyanosis of the oral area and extremities What is the 5 minute Apgar score for this infant? Answer 4 5 6 7

dehydration---

A newborn with gastroschisis is most at risk for which of the following complications? Answer hyperthermia seizures hyperglycemia dehydration

1/5 the weight of the baby

A normal placenta weighs approximately: Answer 1/2 the weight of the baby 1/5 the weight of the baby 3/4 the weight of the baby 7/8 the weight of the baby

partial previa

A placenta that implants low and covers about half of the cervical os is called a: Answer complete previa partial previa marginal previa low-lying placenta

low-lying placenta

A placenta that implants low but covers none of the cervical os is a: Answer complete previa partial previa low-lying placenta marginal previa

antibodies coating the baby's red blood cells

A positive direct Coombs test done on the cord blood indicates the presence of: -antibodies coating the baby's red blood cells -antigens coating the mother's red blood cells -fetal red blood cells in the maternal serum -maternal red blood cells in the fetal circulation

-chorioamnionitis

A possible reason for induction of labor would be: Answer active herpes simplex lesions chorioamnionitis placenta previa transverse fetal lie

vulvarhematoma

A postpartum patient reports she feels very uncomfortable sitting and has a "full feeling down there". The midwife would suspect: -cystitis -a suture may have come undone -postpartum infection -vulvarhematoma

-perform a vaginal exam to feel for water coming from the cervix

A pregnant woman describes some leaking of clear fluid. In order to verify this, the midwife does all of the following, except: -obtain fluid by sterile speculum exam from the cervix -avoid doing a vaginal exam -perform a vaginal exam to feel for water coming from the cervix -take a small piece of nitrazine paper and touch it to the fluid on her pad to confirm ROM

riboflavin and vitamin A

A pregnant woman drinks 1 quart of milk daily, which supplies her with large amounts of: Answer vitamin B12 and niacin folic acid and thiamine riboflavin and vitamin A vitamin C and complete protein

"You should avoid douching in pregnancy, but tell me if the discharge is irritating or excessive"

A pregnant woman says to her midwife, "I usually douche after my period. Is it all right to douche now that I have this discharge?" The most appropriate response by the midwife would be: Answer "You should avoid douching in pregnancy, but tell me if the discharge is irritating or excessive" "You may need to douche more often if you find the discharge offensive" "Douching is prohibited during the second trimester of pregnancy" "Try using some vaginal deodorant spray instead of douching to minimize the drainage"

Harlequin sign

A rare color change that occurs between the longitudinal halves of the newborn's body, such that the dependent half is noticeably pinker than the superior half when the newborn is placed on one side; it is of no pathologic significance.

kernicterus

All of the following are diseases commonly screened for in the newborn during metabolic screening, except: Answer homocystinuria maple sugar urine galactosemia kernicterus

amniocentesis

A technique of prenatal diagnosis in which amniotic fluid, obtained by aspiration from a needle inserted into the uterus, is analyzed to detect certain genetic and congenital defects in the fetus.

rounded inlet, nonprominenant ischial spines and wide, round pubic arch

A typical gynecoid pelvis has which of the following characteristics? -rounded inlet, nonprominenant ischial spines and wide, round pubic arch -heart shaped inlet, prominent ischial spines and a narrow, deep pubic arch -oval outlet, prominent or nonprominent ischial spines, and normal pubic arch -flattened at the inlet, prominent ischial spines and sacrum, and a wide pubic arch

280 days

A typical human gestation (from LMP) is how many days? Answer 280 365 266 245

266 days

A typical human gestation from conception is: Answer 280 days 365 days 266 days 300 days

Do you have any serious hemorrhoids

A woman at 4 days postpartum describes a fever of over 100.4 F for the last 12 hours and general malaise. To determine a differential diagnosis, the midwife would ask all of the following questions, except: - when did your milk come in and are your breasts sore - do you have any serious hemorrhoids - do you have any s/s of a uti - does your lochia smell normal or foul

-to eat frequent, small meals every two hours to prevent hypoglycemia

A woman comes to the midwife in the first trimester complaining of nausea, light-headedness, and lack of appetite that prevents her from eating well. Which of the following best describes the dietary recommendations that the midwife would suggest? Answer -to decrease the food she is consuming until later in pregnancy when the nausea has passed -to eat large meals spaced well apart that contain large quantities of highly concentrated protein -to eliminate all carbohydrates in order to prevent gestational diabetes -to eat frequent, small meals every two hours to prevent hypoglycemia

ectopic pregnancy

A woman goes to the midwife for her initial checkup. She reports feeling ill the day of the appointment. During the routine vaginal exam as part of the initial physical, the midwife notes that she feels tenderness on the right side especially with movement of the cervix, and there is a small mass in the right adnexal area. She does not have a fever, has some nausea but no vomiting, and her last menstrual period was approximately five weeks ago. Which of the following would the midwife suspect? Answer -appendicitis -salpingitis -impending spontaneous miscarriage -ectopic pregnancy

normal postpartum depression or exhaustion

A woman is 6 weeks postpartum and comes to the midwife for her check up. She explains that she has had dyspareunia, is exhausted, is losing weight, and cries often if the house is not clean. The midwife would most likely suspect: - normal postpartum depression or exhaustion - pathological pp psychosis or suicidal depression - OCD - Anorexia nervosa

the baby is presenting shoulder

A woman is in labor. The midwife does Leopold's maneuvers and finds a ballotable prominence in the mother's right lower quadrant, a firm narrow prominence in the pelvis, barely palpable back to the mother's left, firm prominence in the mother's upper left quadrant, and FHT midline below the maternal umbilicus. Which of the following is true? Answer -the baby is presenting shoulder -the baby is presenting LOA -the baby is presenting breech -the baby is presenting ROP

ask for some time alone with her and inquire whether the crowd is bothering her

A woman is laboring at a home, and is acting irritable and cranky with her husband and mother-in-law, who have been with her almost constantly all night. She also has two sisters and a female cousin present, and a doula. The most appropriate action for the midwife to take would be to: - take her on a long walk outside with only her husband and the midwife - ask for some time alone with her and inquire whether the crowd is bothering her - tell everyone to clear the room and stay out until she is pushing because they are in the way

a threatened abortion

A woman reports she is having some slight vaginal bleeding and occasional mild cramps at 10 weeks from LMP. She is best characterized as having: -a threatened abortion -a missed abortion -an imminent abortion -an incomplete abortion

-oliguria

A woman suffering from eclampsia may experience: Answer diuresis oliguria hypotension diabetes

-multipara

A woman who has given birth to three children is a: Answer primipara segunda multipara puerperal

-hemorrhage

A woman with a partial placenta previa delivers following a three hour labor. The complication she is most likely to develop in the early postpartal period is: Answer -hemorrhage -infection -hypofibrinogenemia -prolapsed uterus

soaking in bleach

All of the following are good methods of sterilization for instruments, except: Answer -baking -boiling -pressure cooking -soaking in bleach

-jaundice

All of the following are normal newborn findings in the first 24 hours, except: Answer -enlarged mammary tissue -crystals in the urine -irregular breathing -jaundice

low lying placenta

All of the following are contraindications to labor induction, except: Answer invasive cervical carcinoma unexplained vaginal bleeding low-lying placenta active genital herpes infection

one minute after birth and again at 5 minutes

APGAR scores are done when - immediately after birth and again at 5 min - one minutes after birth and again at 5 minutes - 1 minute after birth and 5 minutes after that - 5 minutes and 10 minutes

Choanal atresia

Abnormality of the nasal septum that obstructs one or both nasal passages.

massage her uterus

About 30 minutes after birth, the midwife notes that Samantha's fundus is boggy and that it is palpable one finger-width above the level of the umbilicus. Her lochia is rubra and heavy. Based on these assessments, the midwife's first action should be to: Answer -obtain a blood sample to assess hemoglobin level -massage her uterus -give her Shepherd's Purse tea -asses her vital signs

-premature separation of the normally implanted placenta

Abruptio placenta is defined as: Answer -abnormal implantation of the placenta -impaired blood supply to the placenta resulting in necrosis -premature separation of the normally implanted placenta -abnormal adherence of the placenta to the uterine wall

lowers the number of placental oxygen exchange sites

Abruptio placenta threatens fetal well being, primarily because it: Answer -increases intrauterine pressure -lowers the number of placental oxygen exchange sites -decreases uterine tone -increases the risk of umbilical cord prolapse

-dispose of contaminated products in a container for recycling

According to O.S.H.A. and the principles of Universal Precautions, the midwife must observe all of the following, except: Answer -wear gloves -wear protective gowns -dispose of contaminated products in a container for recycling -throw used needles in a special sharps container

80-100grams

According to the Brewer Diet and Anne Frye, pregnant women should ideally consume how many grams of protein a day? Answer 10-20 grams 20-40 grams 40-60 grams 80-100 grams

-hemorrhage and need for full newborn resuscitation

After a shoulder dystocia, once the baby is out, which of the following are most likely to occur? Answer -hemorrhage and need for full newborn resuscitation -hemorrhage and need for maternal catheterization -maternal tear and prolonged third stage -fetal sepsis and maternal hypertension

need for rescucitation

After a shoulder dystocia, which is the most likely complication in the baby? Answer hemorrhage in the eyes need for resuscitation sepsis cephalohematoma

contract the uterine muscles

After the birth, Jody is bleeding too much, so the midwife gives 10 units of Pitocin IM. This is indicated to: Answer -contract the uterine muscles -prevent DIC -relax the uterine muscles -augment the clotting action of the platelets

33%

Ainsley is a 24 year old G1 in her 30th week of pregnancy. Which of the following is a normal hematocrit result? - 27% -33% - 37% -40%

-hydrocephalus

All of the following are associated with S.G.A. babies except: Answer -cognitive disabilities -visual and hearing defects -poor infant growth and development -hydrocephalus

estrogen induced relaxation of the large bowel

All of the following contribute to the development of hemorrhoids in pregnancy, except: Answer -estrogen induced relaxation of the large bowel -progesterone induced relaxation of the vein walls -increased pressure of the enlarging uterus causing congestion of pelvic veins -increased pressure of the enlarging uterus leading to increased venous pressure

catheterize her and monitor her urine output

All the following are important steps in evaluating the woman in labor, except: -take vital signs -evaluate the quality, duration and timing of the contractions -catheterize her and monitor her urine output -inspect her ankles, legs and hands for edema

-provides oxygen to the fetus

Amniotic fluid does all of the following except: Answer -absorbs shock -permits fetal movement -maintains fetal temperature -provides oxygen to the fetus

turner syndrome

An example of a sex chromosome abnormality is: -Down's Syndrome -Tay-Sach's disease -Trisomy 13 -Turner Syndrome

-confirmation of Down's Syndrome

An ultrasound at 12 weeks gestation could establish all the following except: Answer -a viable, intrauterine pregnancy -multiple pregnancy -presence of fetal abnormalities such as anecephaly -confirmation of Down's Syndrome

-the study of the structure and shape of the body and the body parts, and their relationship to one another

Anatomy is defined as: -a group of organs that cooperate to accomplish a common purpose -the study of the structure and shape of the body and the body parts, and their relationship to one another -groups of similar cells that have a common function -the study of how the body and its parts work or function

-at the level of the umbilicus

At 20 weeks gestation on an average woman, the top of the fundus should be felt: Answer -1/2 way between the umbilicus and xyphoid process -1/2 way between the pubis and naval -1 finger breadth below the xyphoid process -at the level of the umbilicus

post maturity

Any infant born after 42 weeks

fontanelles

Areas where the neonate's or infant's skull has not fused together; usually disappear at approximately 18 months of age.

level of consciousness

As blood pressure falls during clinical shock, the perfusion of vital organs falls with it; the best indication of whether the brain is being adequately perfused is the: Answer -blood pressure -pulse -level of consciousness -respirations

relaxin

As her pregnancy advances, a woman complains to her midwife that she has developed an awkward waddling gait. Which hormone causes this change? Answer estrogen progesterone relaxin human placental lactogen

a third degree laceration

As the midwife examines a woman's vagina and perineum during the fourth stage of labor, she notices that the woman has a laceration that involves the vaginal mucosa, posterior fourchette, perineal skin, perineal muscles, and the external anal sphincter. This laceration is classified as which of the following: Answer a first degree laceration a second degree laceration a third degree laceration a fourth degree laceration

a breech presentation

As the midwife performs the first of the Leopold's maneuvers on a woman at 38 weeks gestation, she feels a round and hard fetal part in the fundus that is readily moveable and ballottable. This is indicative of which of the following: Answer a breech presentation a cephalic presentation an unengaged presenting part a deflexed attitude

-it is called the Schultz mechanism and results from placental separation that begins centrally

As the placenta is expelled, the midwife notices that the fetal side of the placenta is presenting. Which of the following statements is true about this type of placental presentation? Answer -it is called the Schultz mechanism and results from placental separation that begins centrally -it is the Duncan mechanism and results from placental separation that begins centrally -it is called the Schultz mechanism and results from separation that begins at the margin or periphery of the placenta -it is called the Duncan mechanism and results from a separation that begins at the margin or periphery of the placenta

full set of finger prints

At 15 weeks gestation, the fetus: Answer -could survive outside the womb -is just beginning to grow arm and leg buds -has a full set of fingerprints -has indistinguishable sex organs

dehydration

At a client's one week postpartum check, when the mother is asked about feeding behaviors she tells the midwife, "My baby is incredibly easy. He only nurses for 10 minutes at a time, sleeps for 6 hours between each feeding and he hardly ever poops or pees. I don't have to change his diaper too often." The midwife's first concern is that the baby has: Answer -a short frenulum -dehydration -bacterial meningitis -failure to thrive

5

At one minute after birth, baby Jane is floppy, her extremities are blue, she is breathing shallowly, and her heart rate is 110. Response to stimulus is minimal. Her one minute APGAR would be: -3 -4 -5 -6

6-13 weeks

At the one week checkup, Kendra tells the midwife she cannot breastfeed because she is returning to work. She asks when her periods will return. The midwife states correctly that the non-breastfeeding woman typically resumes menstruating in about: Answer 1-2 Weeks 6-13 Weeks 20-24 Weeks 45-50 Weeks

mastitis---

At two weeks after delivery, Jan develops a small, reddened area on one breast that is hot and tender to the touch. She complains that she feels as if she is coming down with the flu and her temperature is 100.6 F. These signs and symptoms indicate: Answer a blocked milk duct a cracked nipple breast engorgement mastitis

-4-6 months (6 months if the cord was not clamped for at least 5 minutes after birth)

At what age do breastfed infants need to start receiving iron supplementation through food or iron drops? Answer -starting at birth for low birthweight babies -1-2 months for all babies regardless of feeding method -4-6 months (6 months if the cord was not clamped for at least 5 minutes after birth) -12 months for LGA babies

2 weeks

At what week, from conception, does the embryonic period begin? Answer 2 weeks 5 weeks 8 weeks 3 weeks

3rd stage of labor

Delivery of baby --> delivery of placenta

Velamentous insertion

Blood vessels in the umbilical cord separate and leave the cord prior to insertion into the surface of the placenta.

fourchette

The labia minora taper, extending posteriorly to form the: Answer perineal body fourchette perineum posterior commissure

Severe preeclampsia

BP 160/100 or greater, proteinuria greater than 3+, oliguria, elevated serum creatinine- 1.2+, cerebral or visual disturbances (headache or blurred vision), hyperreflexia with ankle clonus, peripheral edema, hepatic dysfunction, RUQ pain, and thrombocytopenia

push the presenting part off the cord

Becky, a 32 year old gravida 2, para 1001, is in strong labor when she arrives at the birth center. The midwife has just taken FHT and found them to be 160 when her membranes rupture suddenly. While looking to assess the amniotic fluid color, the midwife sees the umbilical cord in the introitus. To prevent further complications, the midwife's first action should be to: Answer -call the doctor -help the woman into a high fowler's position -push the presenting part up off the cord -push the cord up into the vagina and hold there while calling 911

milia

Benign, keratin-filled cysts that can appear just under the epidermis and have no visible opening.

acts as a transfer organ for metabolic processes

Besides producing hormones to maintain pregnancy, the placenta: Answer cushions the fetus against pressure maintains the fetus at an even temperature acts as a transfer organ for metabolic processes brings maternal blood directly to the fetus for nutrition

encourage her to push hard and get the baby out fast

Betty is having her 6th baby at home. She has a history of fast labors. The midwife has just checked her and she is 10 cm. With the next contraction the waters break with great force and the cord falls down. She immediately tells the midwife that she has to push. What does the midwife do first? Answer -give her shepherds purse to maximize oxygenation -encourage her to push hard and get the baby out fast -put her in knee and chest position -slow the birth down by pressing on her perineum so that she does not tear

Kernicterus

Bilirubin encephalopathy, a form of brain damage resulting from unconjugated bilirubin entering the brain. Characterized by lethargy, poor feeding, vomiting, irregular respiration, perhaps death

Prematurity

Birth before 37 weeks gestation

post partum hemorrhage

Bleeding in excess of 500cc after delivery. FUNDAL MASSAGE and put infant to breast. Place pad over vaginal opening and treat for shock, elevate hips.

-feelings of fatigue

Bonnie is an 18 year old single mother who is 3 days postpartum. The midwife is worried that she may have postpartum blues. Which of the following is not a sign of postpartum blues in a newly delivered mother? Answer -feelings of being overwhelmed -feelings of being afraid she will harm the baby -feelings of fatigue -feelings of being unable to cope

head compression

Bradycardia in a fetus is often caused by: Answer -head compression -breech presentation -transverse lie -maternal pulse above 90

alveoli

Breastmilk is produced in the: Answer alveoli fatty tissue lymphatic system areola

bluish white and tthin

Breastmilk, while feeding a term infant who is two weeks old, typically looks: Answer yellow and thick bluish white and thin pinkish white and thick yellow and thin

11g/dL

Bree is a 26 year old G1 in her 8th week of pregnancy. Which of the following is an abnormal hemoglobin test result? - 11g/dL -12g/dL -13g/dL -14g/dL

the pigmentary changes that occur during pregnancy

Chloasma is a word used to describe: Answer -the pigmentary changes that occur during pregnancy -a skin condition in the newborn -respiratory difficulties in the newborn -a fungal infection on the skin

natural birth control methods

Calendar/Rhythm Method, Symptothermal, and Standard Day/Bead

polyhydramnios

Candie is a 28 year old G3 P2002 at approximately 32 weeks gestational age. During her routine prenatal check up she is measuring 36 cm. The midwife has difficulty palpating fetal position and finds it very hard to hear fetal heart tones with her fetoscope. The midwife finally hears FHTs with a doppler after having listened all around. Based on this information, the midwife most suspects which of the following: Answer multiple gestation pre-eclampsia polyhydramnios macrosomia

at the level of the pelvic inlet

Case Study: Ophelia is gravida 2 para 0010, with contractions 5 to 8 minutes apart, lasting 40-50 seconds. A vaginal exam shows her to be 5 cm dilation, 80% cervical effacement, -3 station, with ruptured membranes. The presentation is vertex, fetal position is LOA. Based on Ophelia's initial assessment findings, the fetal presenting part is: Answer -at the level of the pelvic inlet -at the level of the ischial spines -1 cm below the ischial spines -at the perineum

transport to a hospital for delivery

Carmen is an Rh negative woman in the first stage of labor who has started showing signs of a sinusoidal fetal heart tone pattern. What should the midwife do? -change maternal position and continue monitoring FHT -give an herbal tincture of Black and Blue Cohosh -transport to a hospital for delivery -give oxygen to the mother by nasal cannula

gravida 3 para 1201

Case Study: Abby, a 26 year old mother of 5 year old twin sons, is making her initial visit to the prenatal clinic. Her obstetric history reveals the birth of a stillborn infant in the 38th week of gestation and the birth of her twins in the 35th week of gestation. She is currently 16 weeks gestation. Based on Abby's obstetric history, the midwife should classify her as: Answer gravida 2 para 1102 gravida 3 para 1201 gravida 3 para 1102 gravida 4 para 1202

normocytic anemia

Case Study: Beth, a 33 year old G4P3 at 26 weeks gestational age, was in a car accident in which she sustained significant blood loss. She is in stable condition now. Her laboratory results reveal the following: Hemoglobin 9 ml/dl; elevated reticulocyte count; and a mean corpuscular volume of 86. These results suggest which of the following: Answer -macrocytic anemia -microcytic anemia -normocytic anemia -hemolytic anemia

respiratory alkalosis

Case Study: Cecille is panting with contractions and complains of tingling and numbness in her hands and feet. These symptoms indicate: Answer respiratory alkalosis metabolic alkalosis respiratory acidosis metabolic acidosis

15 and 20 weeks

Case Study: Connie is pregnant for the first time at age 35. She asks the midwife to order a maternal serum alpha-fetoprotein (MSAFP) screening because of a history of neural tube defects in her family. Connie's MSAFP screening should be performed between what weeks of gestation? Answer 6 and 10 weeks 11 and 14 weeks 15 and 20 weeks 21 and 25 weeks

"That depends on how well your glucose level is controlled by diet"

Case Study: Cora is an Asian 38 year old gravida 3 para 0111, who was diagnosed with gestational diabetes at 30 weeks. The midwife has arranged to transfer her care to a doctor and will continue to advise her prenatally. Cora asks whether she will have to use insulin. The midwife's best response would be: Answer "Yes, you will definitely need to use insulin to control your glucose level" "No, only people with diabetes mellitus type II need to use insulin; you only need to regulate your diet" "That depends on how well your glucose level is controlled by diet" "No, you'll be able to take another medication to control your glucose level"

"Have you been able to empty your bladder?"

Case Study: Ellie, a 30 year old G6P 2305 at 40 weeks arrives at the birth center in early labor with vaginal bleeding that has already soaked an adult diaper. To make a differential diagnosis between abruptio placenta and placenta previa, the midwife would ask the following questions, except: Answer "What color is the blood?" "Do you feel pain?" "Have you been able to empty your bladder?" "Do you have a previous ultrasound showing where the placenta is implanted?"

hypertonic

Case Study: Frieda is a gravida 1 para 0 who has been in labor for 4 hours without any progress. Initial assessment reveals dilation of 2 cm, 100% effacement, -1 station, contractions intensely painful occurring every 2-3 minutes lasting 70 seconds, membranes intact, and FHT of 166. Maternal vital signs include blood pressure of 120/60, temperature 98.6, pulse rate of 94, and respiratory rate of 24. Which contraction pattern is indicated by the above assessment findings? Answer normal hypotonic hypertonic tetanic

collect a clean catch specimen

Case Study: Gabriela is a 22 year old gravida 1 para 1001 who complains of frequent, urgent and painful urination two days after delivery. Her temperature is 101.4 F, she has suprapubic tenderness and moderate lochia rubra. The midwife's first action should be to: Answer -administer 650 mg of Tylenol -assess Gabriela's fundal height and consistency -palpate Gabriela's bladder -collect a clean catch urine specimen

his occupation as a truck driver for five years

Case Study: Janet and Robert are in their late 20's and have been married for four years. They tell the midwife they have never used birth control and want to have a child. Which assessment finding in Robert's history might indicate the cause of the infertility? Answer -his age and length of marriage -his occupation as truck driver for five years -immunization against mumps as a child -basketball playing during his leisure hours

prevent additional thrombus formation

Case Study: Karen is a 35 year old, gravida 5, para 4010 who complains of severe pain and edema of her right leg and thigh on the second day after delivery. A physician consult confirms the diagnosis of femoral thrombophlebitis and the doctor admits Karen to the hospital and orders 5,000 units of heparin every 12 hours. The physician has prescribed heparin for Karen to: Answer -increase the amount of active plasmin in the blood -cause the total lysis of the clot -prevent additional thrombus formation -decrease the risk of developing DIC

gravida 3 para 1011

Case Study: Kim, Hispanic, age 25, comes for an initial visit. She has missed two menstrual periods; her last period was October 30th. Her obstetric history reveals one elective abortion and the birth of a son 4 years ago at 39 weeks gestation. Her present pregnancy has been confirmed. Her husband accompanies her. Based on the above history, the midwife should classify Kim as: Answer gravida 2 para 1101 gravida 2 para 1011 gravida 3 para 0111 gravida 3 para 1011

history of diabetes and familial history of PIH

Case Study: Rene is a 23 year old African American woman, G1P0 at 36 weeks gestation that the midwife meets while volunteering at the local hospital. She has class D diabetes, and today she complains of headaches and increasing finger and ankle edema over the past week. Her blood pressure is 150/110. Rene has just been put on magnesium sulfate therapy by the OB doctor at the hospital for PIH. Rene is with her mother, who tells the midwife she had similar problems during her first pregnancy. Which factors in Rene's health history predisposed her to developing pregnancy induced hypertension (PIH)? Answer -age and race -age and history of diabetes -history of diabetes and familial history of PIH -race and gestational age

"I'd be happy to spend time helping you; what do you want to know?"

Case Study: Rhonda is a gravida 2, para 2002 who has just had a homebirth and plans to breastfeed this baby though she bottle fed her first child. Because she did not breastfeed her first baby, Rhonda now nervously asks for help. The midwife's best response would be: Answer "Breastfeeding is natural; your nervousness will pass with practice" "Let me bring you a video on breastfeeding; it will answer your questions" "I'd be happy to spend time helping you; what do you want to know?" "What made you choose breastfeeding over bottle-feeding?"

hyperemesis gravidarum

Case Study: Sage, a 24 year old gravida 2 para 0010, arrives for a prenatal appointment during her 8th week of pregnancy. She states, "I've been vomiting for 4 days." Vital signs reveal a temperature of 99.4 F, a pulse rate of 100 bpm, a respiratory rate of 24, and a blood pressure of 98/60. Urine tests reveal a high ketone level. Sage's vomiting, vital signs, and ketone level indicate: Answer salmonellosis acute cholecystitis hyperemesis gravidarum a normal early pregnancy

-- GYNECOID

Case Study: The following are the findings of clinical pelvimetry performed on Diana, a 30 year old G1P0:Inlet - anterior, lateral, and posterior segment are well rounded; transverse diameter about the same as the anterioposterior diameterSacrum - parallel with the symphysis pubisSacrosciatic notch - well roundedSidewalls - straightIschial Spines - blunt, non-encroachingPubic Arch - approximately 90 degrees Based on this information, which of the following best describes Diana's type of pelvis: Answer android anthropoid gynecoid platypelloid

blue or purple coloration of the cervix and vulva

Chadwick's sign is: Answer -softening of the cervix -blue or purple coloration of the cervix and vulva -soft spot in the middle of the uterus -enlargement of the uterus

partruition

Childbirth is also called: Answer maturation parturition gestation matriarchism

less fat and carbohydrates

Compared with mature breastmilk, colostrum contains: Answer -less fat and carbohydrates -less protein and minerals -less immunoglobins -the same amount of nutrients

during the 5th month of pregnancy

Congenital syphilis may be contracted by the baby: -at birth -during the fifth month of pregnancy -during the first month of pregnancy -during breastfeeding

"How can I help you process the difficult births you have had?"

Continuation of Case Study: Abby, a 26 year old mother of 5 year old twin sons, is making her initial visit to the prenatal clinic. Her obstetric history reveals the birth of a stillborn infant in the 38th week of gestation and the birth of her twins in the 35th week of gestation. She is currently 16 weeks gestation. Abby expresses fear due to her difficult OB history. Which statement by the midwife is most helpful? Answer "The baby you lost is in heaven; don't feel sad" "Giving birth to twins vaginally is a super feat; you should be proud" "Is there something you would like professional counseling with?" "How can I help you process the difficult births you have had?"

-assess her activities and dietary intake

Continuation of Case Study: Abby, a 26 year old mother of 5 year old twin sons, is making her initial visit to the prenatal clinic. Her obstetric history reveals the birth of a stillborn infant in the 38th week of gestation and the birth of her twins in the 35th week of gestation. She is currently 16 weeks gestation. During the initial assessment, Abby says she has experienced excessive fatigue for the past 3 months. The midwife's initial action should be to: Answer -reassure her that this is a normal discomfort during early pregnancy -suggest frequent rest periods -suggest that she get help in caring for the twins -assess her activities and dietary intake

iron supplementation

Continuation of Case Study: Beth, a 33 year old G4P3 at 26 weeks gestational age, was in a car accident in which she sustained significant blood loss. She is in stable condition now. Her laboratory results reveal the following: Hemoglobin 9 ml/dl; elevated reticulocyte count; and a mean corpuscular volume of 86. Which of the following is the most appropriate treatment for Beth's anemia at this time: Answer -folic acid supplementation -iron supplementation -packed red blood cell transfusion -vitamin B12 supplementation

telling her to exhale into her cupped hands and then reinhale

Continuation of Case Study: Cecille is panting with contractions and complains of tingling and numbness in her hands and feet. Which action would best alleviate Cecille's complaints? Answer -administering 4 liters oxygen by face mask -telling her to breath deeply from her stomach -telling her to exhale into her cupped hands and then re-inhale -having her exhale into a paper bag then breath in room air

myocardial defects

Continuation of Case Study: Connie is pregnant for the first time at age 35. She asks the midwife to order a maternal serum alpha-fetoprotein (MSAFP) screening because of a history of neural tube defects in her family. Connie's MSAFP level is above normal. Which of the following anomalies is not a possible cause? Answer neural tube defects multiple gestation fetal demise myocardial defects

cardiovascular

Continuation of Case Study: Ellie, a 30 year old G6P 2305 at 40 weeks arrives at the birth center in early labor with vaginal bleeding that has already soaked an adult diaper. Which of Ellie's body systems should the midwife assess first, while preparing to transport? Answer endocrine cardiovascular neuromuscular renal

offer her fruit juices along with a high fiber snack

Continuation of Case Study: Frieda is a gravida 1 para 0 who has been in labor for 4 hours without any progress. Initial assessment reveals dilation of 2 cm, 100% effacement, -1 station, contractions intensely painful occurring every 2-3 minutes lasting 70 seconds, membranes intact, and FHT of 166. Maternal vital signs include blood pressure of 120/60, temperature 98.6, pulse rate of 94, and respiratory rate of 24. Frieda states she feels constipated and bloated; which action would be appropriate? Answer -administer 2 fleet enemas with her lying on her side -offer her a full soapy enema with a bag and tubing -offer her fruit juices to drink along with a high fiber snack -give her nothing-by-mouth because she is high risk to have a cesarean

take her to the hospital for oxytocin augmentation

Continuation of Case Study: Frieda is a gravida 1 para 0 who has been in labor for 4 hours without any progress. Initial assessment reveals dilation of 2 cm, 100% effacement, -1 station, contractions intensely painful occurring every 2-3 minutes lasting 70 seconds, membranes intact, and FHT of 166. Maternal vital signs include blood pressure of 120/60, temperature 98.6, pulse rate of 94, and respiratory rate of 24. In making a care plan for Frieda, things the midwife should not consider: Answer -put her in a bath to relax her and slow down contractions -take her to the hospital for an oxytocin augmentation -take her to the hospital for a short-acting barbiturate -hydrate her with fluids and wait a while to see if the pattern will resolve

trauma to the trigone of the bladder

Continuation of Case Study: Gabriela is a 22 year old gravida 1 para 1001 who complains of frequent, urgent and painful urination two days after delivery. Her temperature is 101.4 F, she has suprapubic tenderness and moderate lochia rubra. Lab results reveal a urinary tract infection. The midwife knows the Gabriela may be predisposed to such infections because of: Answer her gravity trauma to the trigone of the bladder her parity weakness of the urethral sphincter

cover her with a blanket and turn up the heat

Continuation of Case Study: Gloria is a 24 year old gravida 1 who has just delivered her first baby without complications. About 15 minutes after delivery, Gloria begins to complain about chills. Her husband worries that she has a fever. What would the midwife do first: Answer -call and consult with a doctor -cover her with a blanket and turn up the heat -administer acetaminophen (Tylenol) -administer aspirin

a firm fundus

Continuation of Case Study: Gloria is a 24 year old gravida 1 who has just delivered her first baby without complications. After completion of the third stage of labor, Gloria continues to bleed. The midwife administers 10 units of oxytocin (Pitocin). Which finding would indicate that the drug has produced the desired therapeutic effect? Answer -a constant hard uterine contraction -a firm fundus -lochial flow turning from bright red to brownish red -expulsion of the intact placenta

maintaining Karen on strict bed rest, monitoring pedal pulses and applying moist heat to the affected area

Continuation of Case Study: Karen is a 35 year old, gravida 5, para 4010, who complains of severe pain and edema of her right leg and thigh on the second day after delivery. Additional therapy for Karen, which the midwife can help with, should include: Answer encouraging walking and exercise of the affected leg maintaining Karen on strict bed rest, monitoring pedal pulses and applying moist heat to the affected area massaging the affected area frequently and applying moist heat maintaining Karen on strict bedrest and applying ice packs to the affected area

august 6

Continuation of Case Study: Kim, Hispanic, age 25, comes for an initial visit. She has missed two menstrual periods; her last period was October 30th. Her obstetric history reveals one elective abortion and the birth of a son 4 years ago at 39 weeks gestation. Her present pregnancy has been confirmed. Her husband accompanies her. Using Naegele's rule, the midwife would calculate Kim's expected date of delivery as: Answer May 2 July 6 July 31 August 6

softening of the lower uterine segment

Continuation of Case Study: Kim, Hispanic, age 25, comes for an initial visit. She has missed two menstrual periods; her last period was October 30th. Her obstetric history reveals one elective abortion and the birth of a son 4 years ago at 39 weeks gestation. Her present pregnancy has been confirmed. Her husband accompanies her. Which assessment finding noted during the exam would best support the diagnosis of pregnancy? Answer -ballottement of the uterus -abdominal enlargement -bluish discoloration of the vaginal discharge -softening of the lower uterine segment

left lower quadrant

Continuation of Case Study: Ophelia is gravida 2 para 0010, with contractions 5 to 8 minutes apart, lasting 40-50 seconds. A vaginal exam shows her to be at 5 cm dilation, 80% cervical effacement, -3 station, with ruptured membranes. The presentation is vertex, fetal position is LOA. The FHT should be most audible in which quadrant of Ophelia's abdomen? Answer -left upper quadrant -left lower quadrant -right upper quadrant -right lower quadrant

-3 station

Continuation of Case Study: Ophelia is gravida 2 para 0010, with contractions 5 to 8 minutes apart, lasting 40-50 seconds. A vaginal exam shows her to be at 5 cm dilation, 80% cervical effacement, -3 station, with ruptured membranes. The presentation is vertex, fetal position is LOA. The midwife decides to keep Ophelia in bed to labor; which assessment finding necessitates this action? Answer 5 cm dilation 80% effacement contractions 5-8 minutes apart -3 station

-establish a patent airway and turn her on her side to prevent aspiration in case vomiting should occur

Continuation of Case Study: Rene is a 23 year old African American woman, G1P0 at 36 weeks gestation that the midwife meets while volunteering at the local hospital. She has class D diabetes, and today she complains of headaches and increasing finger and ankle edema over the past week. Her blood pressure is 150/110. Rene has just been put on magnesium sulfate therapy by the OB doctor at the hospital for PIH. Rene is with her mother, who tells the midwife she had similar problems during her first pregnancy. Rene suddenly experiences a seizure while the midwife is alone with her in the hospital room. The first action when it ends should be to: Answer -establish a patent airway and turn her on her side to prevent aspiration in case vomiting should occur -administer an antihypertensive agent if she is authorized to do so, to prevent a cerebrovascular accident -begin chest compressions and call for assistance with CPR -assess the fetal heart rate for signs of fetal hypoxia and keep an eye on her pulse while waiting for the doctor to arrive

Epigastric pain

Continuation of Case Study: Rene is a 23 year old African American woman, G1P0 at 36 weeks gestation that the midwife meets while volunteering at the local hospital. She has class D diabetes, and today she complains of headaches and increasing finger and ankle edema over the past week. Her blood pressure is 150/110. Rene has just been put on magnesium sulfate therapy by the OB doctor at the hospital for PIH. Rene is with her mother, who tells the midwife she had similar problems during her first pregnancy. Which assessment finding in Rene might be a prodromal manifestation of eclamptic seizures? Answer loss of deep tendon reflexes dramatic decrease in blood pressure epigastric pain respiratory distress

avoid a high protein intake

Continuation of Case Study: Sage, a 24 year old gravid 2 para 0010, arrives for a prenatal appointment during her 8th week of pregnancy. She states, "I've been vomiting for 4 days." Vital signs reveal a temperature of 99.4 F, a pulse rate of 100 bpm, a respiratory rate of 24, and a blood pressure of 98/60. Urine tests reveal a high ketone level. Sage's vomiting subsides and she is able to tolerate fluids. The midwife should not instruct her to: Answer -eat small, frequent meals -drink fluids at least 1 hour after meals -avoid spicy or fried foods -avoid a high protein intake

-dilation of cervix at approximately one centimeter per hour after active labor has begun

Continuation of Case Study: The following are the findings of clinical pelvimetry performed on Diana, a 30 year old G1P0:Inlet - anterior, lateral, and posterior segment are well rounded; transverse diameter about the same as the anterioposterior diameterSacrum - parallel with the symphysis pubisSacrosciatic notch - well roundedSidewalls - straightIschial spines - blunt, non-encroachingPubic arch - approximately 90 degrees Which of the following labor patterns would the midwife predict for Diana based on her pelvimetry: Answer -long difficult labor with posterior position of fetal head -deep transverse arrest in second stage -dilation of cervix at approximately one centimeter per hour after active labor has begun -fetal head will remain high and unengaged in spite of strong labor contractions close together

"Breastfeeding causes the uterus to contract; you can take Tylenol if needed"

Continued Case Study: Rhonda is a gravida 2, para 2002 who has just had a homebirth and plans to breastfeed this baby though she bottle fed her first child. At the 24 hour visit, Rhonda tells the midwife that breastfeeding makes her cramping get much worse, and she really wants relief from the pain even if she has to give up breastfeeding. The midwife should respond by saying: Answer "Breastfeeding causes the uterus to contract; you can take Tylenol if needed" "Your technique may be causing this problem; let me watch you nurse "The cramping is normal, but you can't take any medication while nursing" "The cramping will pass; just continue breastfeeding and don't worry"

----uterine involution

Continued Case Study: Rhonda is a gravida 2, para 2002 who has just had a homebirth and plans to breastfeed this baby though she bottle fed her first child. Rhonda complains of painful cramping each time she breastfeeds. The midwife knows such cramping is commonly caused by: Answer wrong positioning of the baby to the breast uterine sub-involution uterine involution bladder distention

-leakage of milk from one breast while the baby nurses at the other breast

Continued Case Study: Rhonda is a gravida 2, para 2002 who has just had a homebirth and plans to breastfeed this baby though she bottle fed her first child. Which sign indicates that Rhonda's let-down reflex is functioning properly? Answer -the baby's vigorous sucking at the breast -nipple soreness -leakage of milk from one breast while the baby nurses at the other breast -a feeling of breast fullness

increase when walking

Contractions that are a sign of active labor will: Answer -mainly be felt in the front -decrease when walking -increase when walking -be irregular

autosomal recessive disorders

Cystic fibrosis, sickle cell anemia, and Tay-Sachs disease, all of which can be diagnosed by prenatal genetic testing, are examples of: Answer -autosomal dominant disorders -X-linked recessive disorders -autosomal recessive disorders -X-linked dominant disorders

an expanded plasma volume

Deborah is a 30 year old G2 P 1001, Caucasian at 30 weeks gestation. The midwife sees her for a routine prenatal in her home, where she tests for hemoglobin and finds that Deborah's levels have fallen to 11.2 from her previous pregnant level of 14.5. This decrease is probably the result of: Answer -increased fibrinogen levels -iron deficiency anemia -an expanded plasma volume -a low vitamin C intake

scarf sign

Drawing an arm across the chest toward the newborn's opposite shoulder until resistance is met. Preterm will cross the midline of the chest, full term will not cross midline.

oliguria

Decreased urine output

herpes simplex virus

Darla comes to the midwife complaining of painful and "itchy" bumps on her vagina. Upon examination the midwife notes a half dozen vesicles on her labia minora and majora. Darla states that she has had a fever and general malaise for about 5 days. She has some inguinal lymphadenopathy. Which of the following is the most likely cause of these findings? Answer secondary syphilis herpes simplex virus human papillomavirus bartholin's cysts

-discuss her concerns with the mother's backup physician as to how to best proceed

Despite the midwife's best efforts to give care to a mother who is a survivor of sexual abuse, she has continued to have severe emotional problems. The midwife questions the wisdom of a home birth for her. Before jumping to conclusions, the midwife would: Answer -talk to her about her issues and tell her that she has been a bit much to handle and may need to consider a hospital birth -discuss her concerns with the mother's backup physician as to how to best proceed -tell the mother that she will overlook these issues and proceed with the plans for a home birth if the woman pays on time -tell her most women with previous sexual abuse have these emotional problems and should consider a cesarean

popliteal angle

Determined with the newborn flat on his back, flex the thigh on the abdomen and chest and extend the lower leg till resistance is met, measure angle.

cortex of the ovaries

Developing follicles and the graafian follicle are found in the: Answer -medulla of the ovaries -cortex of the ovaries -infundibulum -mesovarium ligament

placenta succenturiate

Development of placental tissue separate from the main body of the placenta; accessory placenta

-compression of the urethra

Difficulty in voiding after childbirth may be due to all the following factors, except: -decreased sensitivity of the bladder -lack of fluid intake during labor -relaxation of the abdominal wall -compression of the urethra

vertical transmission

Disease that passes from parent to offspring across the placenta is transmitted by a: -horizontal spread -vertical spread -circular spread -oblique spread

bleeding of the nose and gums

Disseminated intravascular coagulation (DIC) is a possible complication of abruptio placenta. Which assessment finding would indicate that a woman is developing DIC? Answer -bleeding of the nose and gums -rapid clotting at the site of the IV stick -joint pain -increased pain and anxiety

this is normal and common

During a breast exam at the initial prenatal visit, the midwife observes that one breast is slightly smaller than the other. She would explain to the woman that: Answer -this is normal and common -this is indicative of a tumor -this is unusual and rarely seen, but nothing to worry about -she should consult a lactation consultant because it may cause problems with breastfeeding

"No level of alcohol intake has been proven safe during pregnancy"

During a childbirth class, a pregnant woman asks the midwife if she can have wine occasionally. The midwife's best response would be: Answer "Any alcohol is unsafe during the first trimester, but an occasional glass after that is ok" "A minimal alcohol intake is safe throughout pregnancy" "Alcohol, consumed even in minimal amounts, is associated with LBW babies" "No level of alcohol intake has been proven safe during pregnancy"

"I always lie down on my left side for about one hour after eating a big meal"

During a prenatal visit at 35 weeks gestation, a client complains of heartburn and receives instructions on relieving it. Which statement by the client would indicate the need for further teaching on the part of the midwife? Answer "I am avoiding sugary or fatty foods" "I always lie down on my left side for about one hour after eating a big meal" "I'm now baking or broiling meats instead of frying them" "I'm drinking skim or low fat milk between meals instead of with meals"

---face presentation

During an abdominal examination of a woman at term, the midwife feels the cephalic prominence on the same side as the fetal back. This is indicative of which of the following: Answer -face presentation -vertex (occipital) presentation -brow presentation -sinciput (military) presentation

vertex (occipital) presentation

During an abdominal examination of a woman at term, the midwife feels the cephalic prominence on the same side as the fetal parts. This is indicative of which of the following? - face presentation - vertex (occipital) presentation -transverse lie -ROA Position

--tachycardia

During first stage of labor, a clinical sign of maternal distress is: Answer hypotension tachycardia bradycardia respirations of 16 per minute

in between the nipples

During neonatal CPR, the midwife should place her fingers: Answer in between the clavicles in between the nipples just under the xiphoid process over the rib cage

prolonged with failure of the head to engage

During pelvimetry, a primigravida is found to have a diagonal conjugate of 10 cm. Based on this information, the midwife would expect her labor to be: -within normal length of time for a primigravida -prolonged, with slow fetal descent after +2 (plus two) station -prolonged, with failure of the head to engage -normal in first stage, with prolonged pushing in second stage

the formation of the mucus plug

During pregnancy, the increased number and activity of the endocervical glands are responsible for: Answer -marked softening of the cervix -a thinner, more watery mucosal discharge -the development of Chadwick's sign -the formation of the mucus plug

5 minutes

During second stage, heart tones should be taken every: Answer 2 hours 1 hour 30 minutes 5 minutes

-ask her to describe the amount and odor, and if she has any itching, burning or pain

During the 20th week of gestation, a pregnant woman tells the midwife she has been having "thick, white vaginal discharge". Which assessment procedure should the midwife perform initially? Answer -inquire about the type of underwear or panty hose she has been wearing -determine if she uses hygienic deodorants -ask her to describe the amount and odor, and if she has any itching, burning or pain -assure her that this is a normal change, all pregnant women experience more discharge

hands and knees and back counter pressure

During the active phase of labor, Tesa reports severe back pain that becomes increasingly intense during contractions. The midwife would suggest which of the following: Answer -enema and rebozo -pelvic tilt exercises and warm compresses -hands and knees position and back counter-pressure -side-lying on the side of the fetus's back and moxibustion

-strength of the uterine contractions

During the midwife's initial vaginal exam on a woman in labor, she will be checking for all of the following, except: Answer -location of the cervix -station of the fetal head -condition of the external genitalia -strength of the uterine contractions

imperforate anus

During the newborn exam, the baby has respirations of 80, heart rate of 140, temperature of 96 F, and cyanosis. The cause could be any of the following, except: Answer cold stress meconium aspiration infection imperforate anus

full term

During the newborn exam, the midwife discovers that: the skin is pink, there is a little vernix, and the ears are firm and well formed. The midwife will say that the newborn is: - well formed - full term - post dates - pre term

pressure from the enlarging uterus

During the second trimester, a pregnant woman develops recurrent ankle edema in the afternoon. Assessment reveals that her blood pressure and urinalysis results are within normal limits. The edema is probably related to: Answer -pressure from the enlarging uterus -pregnancy induced hypertension -increased serum albumin levels -increased progesterone levels

investigate further with recent history questions, and discuss with her such things as failure of contraception and the possibility of uterine fibroids

During well woman care, the midwife performs a pelvic exam and finds that a woman's uterus is easily palpable and somewhat irregular above the pubic bone, indicating possible enlargement. The most appropriate response to this finding would be: Answer -trust her insistence that there is "no way" she could be pregnant, decide it must be normal for her, and move on with the exam -draw blood for a pregnancy test and run a prenatal panel with CBC and give her all the handouts for early pregnancy -investigate further with recent history questions, and discuss with her such things as failure of contraception and the possibility of uterine fibroids -send her immediately for an ultrasound to rule out cancer of the uterus

2-12 weeks

During what period of pregnancy is the baby most susceptible to teratogens (from conception)? Answer 1-4 weeks 16-20 weeks 2-12 weeks 36-40 weeks

----- breech presentation

Esther is in labor with cervical dilation of 4 cm, 100% effacement, 0 station, contractions moderately intense and occurring every 5 to 6 minutes and lasting 45-60 seconds, membranes intact, FHT are loudest in the upper left quadrant, FHT is 140-150 bpm with normal variability. Maternal vital signs include blood pressure of 124/70, pulse rate of 84 bpm, respiratory rate of 20, and temperature of 99. When performing Leopold's maneuvers the midwife detects a hard, round object at the level of the fundus. Assessment findings for Esther indicate that the fetus is in a: Answer breech presentation cephalic presentation posterior position transverse lie

normal labor

Eutocia is defined as: Answer -a difficult or obstructed labor -the thinning of the cervix -a normal labor -the degree of fetal descent

sacrum

Each innominate bone of the pelvis is comprised of these parts, except for the: Answer ischium illium pubis sacrum

acme

Each uterine contraction consists of three phases. The contraction is the strongest during which phase? Answer increment acme climax decrement

--head compression

Early decelerations are thought to be caused by: Answer -head compression -uteroplacental insufficiency -cord compression -first stage

thinning of the cervix

Effacement is defined as the progressive: Answer dilation of the cervix thinning of the cervix degree of descent strengthening of contractions

Square window sign

Elicited by gently flexing the newborns hand toward the ventral forearm until resistance is felt. The angle formed at the wrist is measured

dilation

Enlargement of the external cervical os to 10 cm in diameter is called: Answer effacement dilation descent crowning

-candida albicans

Excessive use of antibiotics may result in a vaginal infection with: -Candida albicans -Human immunodeficiency virus -Bacterial vaginosis -Genital herpes virus

Kneeling Breech presentation

Extension at thighs, flexion at knees, Knees present

sharp protuberant nose

FAS often present with characteristic physical features. which of the following is not a common features associated with FAS? - short eye openings - sharp protuberant nose - narrow lip with indistinct center groove - smaller than normal head with dysmorphic features - abnormal testicles in males

Mild bradycardia

FHR 100-120 bpm - Often seen in a postmature fetus

maternal sleep

FHR variability is not affected by: - maternal sleep - the second stage of labor - fetal sleep - maternal drug use

septic shock

Failure of tissue perfusion caused by changes in the circulation that occur as a result of severe infection is called: Answer metabolic shock septic shock neurogenic shock anaphylactic shock

"It is still too early to feel movements; you should begin to feel them in 4-6 weeks"

Felicity is a 20 year old primigravida woman making her initial visit to the midwife at 14 weeks gestation. Felicity mentions to the midwife, "I am worried -- I haven't felt any movement yet." The most appropriate response by the midwife would be: Answer "You better set up a physician consult and mention this to your doctor" "It is still too early to feel movements; you should begin to feel them in 4-6 weeks" "Because this is your first pregnancy, you may not recognise fetal movements. If you fail to feel these movements within another week, let me know" "Some babies are just quieter in the womb than others"

"Your body is ridding itself of some of the excess fluid from pregnancy"

Grace Ann tells the midwife the day after delivery, "When I woke up, my nightgown was soaked with perspiration. Is this normal?" The correct response would be: Answer "Yes, it is normal; you probably have a slight fever" "Sweating at night can be a sign of infection; let's do a blood test" "Sweating at night is abnormal; be sure to tell your doctor" "Your body is ridding itself of some of the excess fluid from pregnancy"

the syncytiotrophoblast

HCG is secreted in which of the following locations - the syncytiotrophoblast - the oocyte - the ovary - the thymus

-ask her to come in right away so that she can have a CVAT assessment and do a urine test to rule out a UTI

Hailey is a 16 year old in her 36th week of her first pregnancy. She calls to cancel her prenatal appointment with the midwife, saying she has come down with the stomach flu her family had the previous week. Her symptoms include backache, vomiting, and fever of 102 F. The midwife should: Answer -tell her these are very serious symptoms of pre-eclampsia and she should come in and be checked for hyperreflexia -recommend that she take ginger to alleviate the vomiting and use a hot pack on her back, or schedule a massage and come in next week -ask her to come in right away so that she can have a CVAT assessment and do a urine test to rule out a UTI -tell her to feel better soon and to remember to drink a lot of fluids and get rest before labor begins

check the FHT

Helen is a gravida 2 para 1001 who calls the midwife to her home reporting contractions every 3 minutes, lasting 50-60 seconds and says her water just broke. The midwife's first action upon arrival should be to: Answer -check the FHT -call for her backup midwife -check the vaginal discharge with nitrazine paper -perform a sterile vaginal exam to rule out prolapsed cord

rh- mother with Rh+ father

Hemolytic disease of the neonate may be produced by the union of: Rh- mother with Rh- father Rh- mother with a Rh+ father Rh+ mother with Rh-father type B mother with type O father

500cc

Hemorrhage is said to take place when the blood loss exceeds: Answer 300 cc 100 cc 500 cc 2,000 cc

6 weeks

How long after birth does complete regeneration of the endometrium at the placental site take? Answer 1 week 2 weeks 4 weeks 6 weeks

46

How many chromosomes are there in the human cell? Answer 4 12 23 46

The syncytiotrophoblast

HpL is primarily secreted from what location? - The syncytiotrophoblast - corpus lutem - thyroid - ovary

mild preeclampsia

Hypertension beginning after the 20th week of pregnancy with 1 to 2+ proteinuria and a weight gain of more than 2 kg per week in the second and third trimesters.

-the practitioner causes

Iatrogenic complications are complications that: Answer -arise from root causes -could not have been prevented -the practitioner causes -involve septic infection

"You seem to be uncertain about the role you should be playing at this time"

Ida is a primigravida seeing the midwife for her first visit at 29 weeks gestation. The husband tells the midwife that his wife is insisting he attend childbirth classes with her. He says he does not see why he needs to attend since he read all the books she gave him. Which response by the midwife would be most appropriate? Answer "You should attend the classes because they are very valuable" "Are you afraid to share the experience of childbirth with your wife?" "Don't you want to support your wife during the pregnancy?" "You seem to be uncertain about the role you should be playing at this time"

OA or OP position

If a baby entered the pelvis in the LOP position, to be born he must rotate to a: Answer -mentum anterior position -mentum posterior position -transverse position -OA or OP position

SGA

If a baby has suffered from uteroplacental insufficiency, the midwife can expect to see a baby who is/has: Answer FAS SGA CNS damage LGA

mentum anterior position

If a baby is delivering in face presentation, he must rotate to: Answer -transverse position -oblique position -mentum posterior position -mentum anterior position

irreversible

If a fetus suffers from malnutrition in the womb during the time of development when cells are increasing in number, then the damage suffered by the fetus is best characterized as which of the following? Answer irreversible reversible asymmetric catabolic

25%

If a pregnant woman has been diagnosed with gestational diabetes but is within the normal BMI (her weight is within the ideal range for her height), what are the percentage odds of her developing diabetes mellitus sometime in the future? Answer 0% 5% 25% 60%

toxoplasmosis

If a pregnant woman has cats or eats undercooked meat, she could be at risk for: Answer Hepatitis A Toxoplasmosis Cirrhosis of the liver Pre-eclampsia

28 weeks

If a woman is Rh-negative and unsensitized, approximately when in pregnancy should she receive Rh immune globulin (RhoGAM) to prevent possible Rh sensitization if she so desires? Answer 20 weeks 28 weeks 32 weeks 36 weeks

degeneration of the corpus luteum

If conception fails to occur, menstruation takes place as a result of: Answer -increased progesterone -stimulation of the graafian follicle -Gn-RH secretion -degeneration of the corpus luteum

forego vaginal examinations to prevent introducing any germs; just insert amni-hook in her vagina

If considering an amniotomy, the midwife will do all the following, except: - -use sterile technique and instruments, and only perform if medically indicated -explain the procedure to the mother and obtain her informed consent -perform only if the head is well engaged, and listen to the FHT immediately following the amniotomy -forego vaginal examinations to prevent introducing any germs; just insert amni-hook in her vagina

dyspareunia

If discovered during a prenatal exam, all of the following would necessitate a physician consult, except: Answer -infection -lack of FHT -severe anemia -dyspareunia

elevate the womans legs

If hypovolemic shock is occurring, the midwife should: Answer -elevate the woman's legs -allow her to rest or sleep -give her a drink of water to keep her hydrated -refuse to give her a blanket

get the placenta out

If the client continues to hemorrhage in third stage, the most important thing is to: Answer -apply supra pubic pressure -get the placenta out -give methergine -transport

--Cephalahematoma

If the mother passed a sexually transmitted disease to her baby, the baby may suffer from any of the following, except: Answer pneumonia blindness rashes and tender bumps on the body cephalohematoma

spurt in an arch

If there is arterial bleeding, the blood will: Answer -spurt in an arch -gush then stop right away -trickle bleed and be watery -be thick and deep red

massage the uterus

Immediately after the delivery of the placenta, it is most important to: Answer -check the baby's heart rate -give the mother water to drink -massage the uterus -check maternal blood pressure

zygote

Immediately following fertilization, the fertilized egg is called a: Answer morula zygote embryo blastocyst

-on the anterior or posterior surface of the fundus

Implantation usually occurs: Answer -on the anterior or posterior surface of the fundus -in the fallopian tubes -in the lower uterine segment -in the abdominal cavity

Placenta accreta

Improper implantation of placenta into the myometrium with little or no intervening decidua

-same side as the fetal back

In a face presentation, the higher cephalic prominence will be on the: Answer -same side as the fetal back -opposite side of the fetal back -top of the fundus -not palpable

14

In a normal monthly cycle, ovulation occurs around day: Answer 7 21 14 2

larger than normal

In a pregnancy complicated by hydatidiform mole, the size of the uterus is most often: Answer same as a normal gestation larger than normal smaller than normal normal size for gestation but the uterus is pushed off to one side or the other

eyes

In addition to the urethra, the organisms of gonorrhea may also infect the newborn's: Answer eyes muscles liver kidneys

reducing the time a baby is in distress after reaching the perineum

In an emergency, the midwife may need to perform an episiotomy. This procedure is performed for which of the following indications: Answer -preventing perineal lacerations -avoiding stretching and possibly tearing the perineum -shortening the third stage of labor to get the baby out quickly -reducing the time a baby is in distress after reaching the perineum

90 degrees

In clinical pelvimetry, if the pelvis is adequate, the angle of the pubic arch should be at least: Answer 90 degrees 70 degrees 1 finger breadth 5 finger breadths

the sympto thermal method

In counseling a woman interested in natural family planning, the midwife would inform her that the most effective method is: -the calendar/rhythm method -the basal body temperature method -diaphragm/withdrawal method -the sympto-thermal method

-iron, folate, zinc, calcium, vitamins D, E, and B6

In general, along with adequate food intake, which of the following supplements would be included in a discussion of good nutrition in the pregnant woman? Answer -iron, folate, zinc, calcium, vitamins D, E, and B6 -bioflavonoid, zinc, and sodium -copper, manganese, omega 6 and chelated potassium -hydrochloric acid, various strains of bifidus, and hypericum perforatum

day 6 before birth to day 2 following birth

In which days surrounding childbirth is it most likely that maternal varicella infection will be passed to the newborn? Answer -day 10 before birth to day 5 following birth -day 2 before birth to day 6 following birth -day 6 before birth to day 2 following birth -day 5 before birth to day 10 following birth

the ampulla

In which portion of the fallopian tubes does fertilization normally occur? - the ampulla -the interstitial segment -the isthmus

threatened miscarriage

Indications for a biophysical profile include all of the following, except: Answer threatened miscarriage post dates known or suspected IUGR pre-eclampsia

evaluation of RDA's according to body type, activity level, age, gestational age, and specific individual needs based on increased demand

Individualized nutritional evaluation for each client should include which of the following? Answer -evaluation of correct dietary practices that can be applied to every pregnant woman so that she will fit into a prescribed definition of what constitutes good nutrition -critical investigation that will give credence to the midwife's knowledge of what is best for each person -evaluation of RDA's according to body type, activity level, age, gestational age, and specific individual needs based on increased demand -the prevention of excessive weight gain that can lead to gestational diabetes and hyperemesis gravidarium

respiratory distress syndrome

Infants of diabetic mothers are at risk for which of the following problems? -erythroblastosis fetalis -hypercalcemia -respiratory distress syndrome -hypothyroidism

amnion

Innermost membranous sac surrounding the developing fetus

Oxytocin challenge test (OCT)

Intravenous injection of oxytocin causing the uterus to contract; monitoring of the fetal reaction to uterine contractions aims to determine how the fetus reacts to environmental stress

placenta circumvallata

Placenta with a double fold of amnion forming a ring on the fetal surface some distance in from the edge of the placenta

latent or false labor

Irregular contractions with no dilation are a sign of: Answer -true, or active labor -latent, or false labor -hypertonic labor -hypotonic labor

tylenol

Is tylenol or ibuprophen considered safe during pregnancy? - tylenol - ibuprophen - neither - both

11g/DL

Isa is a 25 year old G1 in her 31st week of pregnancy. which of the following is a normal hemoglobin test result? - 9g/dL - 11g/dL -15g/dL -17g/dL

regular contraction that increase in frequency and duration over time

It is thought that a woman is experiencing true labor when her contraction pattern shows: Answer -occasional irregular contractions -irregular contractions that increase in intensity -regular contractions that remain the same -regular contractions that increase in frequency and duration over time

the neonates first growth spurt

Janine had a baby 10 days ago and during a routine check up, she mentions to her midwife that the baby is requiring more frequent breast feeding. This change is probably associated with: - A diminished milk supply - the neonates first growth spurt - the need for supplemental feedings, such as cereal - early stages of mastitis

determine the cause of bleeding and take vital signs

Josie delivered 15 mina go. The placenta came 5 mina go. Blood loss so far is 1000cc and she still has a trickel bleed. She is showing signs of shock. What should the midwife do first - determine source of bleed and take vitals - give methergine then check BP - treat for anemia with iron supplements and arrange for blood transfusion - keep mother warm and give nothing by mouth

transport

Julie's water has just broken and there is thick meconium. The midwife checks the FHT and they are 80 bpm and do not come back to baseline after a contraction ends. The midwife checks her and finds she is 4 cm and 0 station. She would do which of the following: Answer -transport -stimulate the baby's head -prepare for resuscitation -give the mother oxygen

monitoring her temp every 2 hours

June calls the midwife to her home in the late morning after laboring alone all night. The midwife determines that June's amniotic membranes ruptured several hours ago. Which action would be the midwife's priority in caring for June? Answer -increasing her fluid intake -monitoring her fluid intake and output -cleaning her perineal area often during labor -monitoring her temperature every 2 hours

4th stage of labor

Placental delivery--> maternal stabilization (up to 48hrs for both)

suggest to lacey that she may need more privacy

Lacey's large family is making her nervous and contractions are slowing down. The midwife should: Answer -take Lacey out for a walk to strengthen labor and clear her head -ask the family to all leave the room -suggest to Lacey that she may need more privacy -ignore the family and concentrate on Lacey only

-uteroplacental insufficiency

Late decelerations are thought to be caused by: Answer -compression on the umbilical cord -compression of the head -uteroplacental insufficiency -second stage

a profuse, thin or thick vaginal secretion that begins in the first trimester

Leukorrhea is defined as: -the absence of menstruation -a profuse, thin or thick vaginal secretion that begins in the first trimester -spotting during the pregnancy with or without pain -the presence of leukocytes in the urine

fetal vagal nerve stimulation

Listening through a contraction with the doppler, the midwife notes that a FHT deceleration occurs during the increment of the contraction, reaches its lowest point at the acme of the contraction, and returns to baseline during decrement of the contraction. This type of deceleration indicates: Answer fetal distress uteroplacental insufficiency fetal vagal nerve stimulation umbilical cord compression

screen for gestational diabetes mellitus

Lois is still seeing a doctor for prenatal checkups, along with her midwife, and reports to the midwife that the OB has scheduled her for a 1 hour glucose tolerance test (GTT) during her 28th week of pregnancy. She asks the midwife the reason for the test. The midwife should respond that it will: Answer -determine if she has diabetes mellitus -screen for gestational diabetes mellitus -screen for pregnancy induced hypertension (PIH) -diagnose gestational diabetes so she can adjust her diet

-tell her to breathe by blowing air through her mouth when she feels the urge to push

Lynelle suddenly says, "I'm feeling so much pressure...I want to push." After a few contractions, the midwife checks her cervix and finds that the cervix is dilated 7 cm. How should the midwife respond? Answer -tell her to begin pushing -tell her to breathe by blowing air through her mouth when she feels the urge to push -get ready for imminent delivery -instruct her to hold her breath when she gets the urge to push

----head compression

Maddy is fully dilated and when she begins to push during contractions, the FHT drops to 90 BPM and then quickly returns to baseline when she stops pushing. This sudden change is probably the result of: Answer maternal drug use maternal position abruptio placenta head compression

decrease neuromuscular irritability

Magnesium sulfate is administered primarily to: Answer -increase the central nervous system's response to stimuli -decrease neuromuscular irritability -increase calcium reabsorption by the muscles -reduce peripheral vascular resistance and lower the blood pressure

barrier birth control methods

Male and female condoms; blocks sperm from entering an opening.

"Try telling him stories about his birth and encourage him to listen to the baby's heartbeat"

Maribell tells the midwife she has tried to prepare her 4 year old for becoming a sibling but he does not seem interested. Which response by the midwife would be most helpful? Answer "It's still early. Why not wait until one month before your due date?" "Why have you waited so long? Isn't he curious about your enlarging abdomen?" "He might respond more favorably if your husband takes a more active role" "Try telling him stories about his birth and encourage him to listen to the baby's heartbeat"

heart rate above 180

Marked tachycardia in a fetus is defined as: Answer heart rate above 140 heart rate above 160 heart rate below 120 heart rate above 180

patellar, ankle clonus, and achilles

Maternal baseline reflexes include: Answer -babinski, moro, and plantar -Goodell's, Hegar's, and Chadwick's -pulse, blood pressure, and temperature -patellar, ankle clonus, and achilles

-treponema pallidum

Maternal malnutrition can lead to all of the following conditions, except: Answer -hypertrophic and hyperplastic brain growth -inadequate uteroplacental function -treponema pallidum -IUGR and SGA

-deficiency in folic acid or vitamin B12 malabsorption

Megaloblastic anemia is primarily due to which of the following: Answer -a lack of vitamin C and D in the diet -blood loss or fluid loss from the cells -deficiency in folic acid or vitamin B12 malabsorption -metabolic changes of pregnancy

what questions do you have about adoption, megan?

Megan is a quiet, shy 17 year old with an unplanned pregnancy. Her mother usually accompanies her on her prenatal visits. Today, her mother asks the midwife about adoption resources. The midwife would most correctly respond: Answer "That is a good thing to think about. I will get you a list of local agencies" "How would you like me to help you prepare for releasing your baby for adoption, Megan?" "What questions do you have about adoption, Megan?" "I know a wonderful couple who have a loving home and hope to adopt a newborn; I will get you their phone number"

Bandl's ring

Meisha, a G1P0, has been in hard labor for 24 hours and has reached 7 cm. The midwife notices a horizontal indented line beginning to appear across her abdomen. This is likely: Answer -Bandl's ring -a full bladder -indication that the baby is posterior -this is a normal occurrence at 7 cm

Gravida 4 para 3 AB 0

Molly is newly pregnant. She has two children living at home. A third child died of birth defects at 4 days of age. She has never had an abortion or miscarriage. Which of the following accurately describes her now? Answer Gravida 3 para 2 AB 0 Gravida 3 para 3 AB 0 Gravida 4 para 2 AB 0 Gravida 4 para 3 AB 0

iron deficiency

Most anemia during pregnancy is caused by: Answer parasites iron deficiency sickle cell disease hemorrhage

20th week

Mothers usually feel their baby kick for the first time around the: Answer 10th week 20th week 28th week 30th week

90/50 - 140/90

Normal adult blood pressure for a pregnant woman is: Answer 90/60 - 130/70 90/50 - 140/90 70/40 - 160/75 50/90 - 120/80

-color deteriorates with crying

Normal neonates have the following characteristics, except: Answer -color deteriorates with crying -their color improves with crying -physiologic jaundice is expected on the third and fourth days -pathological jaundice appears within the first 24 hours or persists past the 7th day

Embolism

Obstruction of a blood vessel by a clot of blood or foreign substance

advise her to discontinue breastfeeding until the engorgement goes away

On a two week postpartum check up, evaluation of the mother's breasts show them to be slightly swollen and tender and hot to the touch. The midwife's treatment would include all of the following, except: - assess for further signs of infection - counsel her about position changes during feedings and proper latch - advise her to discontinue nursing until engorgement goes away - consider using heat or massage as comfort measures

LST

On a vaginal exam of a breech presentation, the midwife feels the sacrum in the left, transverse portion of the maternal pelvis. Which of the following is used to describe this fetus' variety and position? Answer LOT LST frank LOT frank LST

-2 finger breadths below the umbilicus

On the first day postpartum, the fundus should be: -at the level of the symphysis pubis -at the level of the umbilicus -midway between the umbilicus and the symphysis pubis -2 finger breadths below the umbilicus

triplex

One placenta divided into three parts, either completely separated or joined in part, but held together by one set of membranes is called a: Answer succenturiate vasa previa triplex duplex

the formation of the ovum

Oogenesis is defined as: Answer the formation of sperm the formation of ovum haploid gametes diploid gametes

-approximately 14 days before the onset of menstruation

Ovulation occurs: Answer -at the time of menstruation -approximately 14 days after the onset of menstruation -approximately 14 days before the onset of menstruation -somewhere around 28 days after the last menstruation

-it decreases as pregnancy progresses

Paula is 41 weeks and very worried about being overdue. She asked the midwife to order an ultrasound for her to determine when her baby will come. Which of the following statements is true regarding the accuracy of ultrasound to determine gestational age? Answer -it increases as pregnancy progresses -it decreases as pregnancy progresses -the stage of pregnancy does not affect accuracy -ultrasound is not an accurate means of determining gestational age

the study of how the body and its parts work or function

Physiology is defined as: -the study of how the body and its parts work or function -a structure composed of two or more tissue types that performs a specific function -the study of the structure and shape of the body and body parts, and their relationship to one another -the smallest of all living units

-the placenta is abnormally placed and partially or totally covers the cervix

Placenta previa is defined as: Answer -the placenta is abnormally placed and partially or totally covers the cervix -the placenta is normally placed but one or more accessory lobes covers the cervix -the placenta grows over the top of a previous cesarean scar -the placenta is low lying in the lower uterine segment

subjects the mother to hemorrhage

Placenta previa threatens maternal well being, primarily because it: Answer -causes pregnancy induced hypertension -subjects the mother to hemorrhage -subjects the mother to the risk of uterine rupture -is known to cause erythroblastosis fetalis

pressure on the lumbosacral nerve plexus

Pregnant women often experience muscle cramps in one or both legs. These cramps may be the result of: Answer -physiological hemodilution -pressure on the lumbosacral nerve plexus -pressure on the pudendal nerve -calcium-potassium imbalance

labor prior to 37 weeks

Preterm labor is defined as: Answer -labor starting after 38 weeks -labor starting prior to 37 weeks -labor starting with premature rupture of membranes -labor starting before 32 weeks

metrorrhagia

Profuse bleeding from the uterus, especially between periods, is called: Answer metrorrhagia amenorrhea dysmenorrhea toxic shock syndrome

morbidity

Refers to ill health in an individual and the levels of ill health in a population or group.

she must avoid pregnancy for at least 3 months

Regina's prenatal lab work showed a negative rubella titer, so the midwife recommends postpartum that she receives a vaccination. Which information should the midwife stress to Regina? Answer -she must avoid pregnancy for at least 3 months -the rubella vaccine has no adverse affects -she should avoid contact with her baby for 24 hours after the vaccination -the rubella vaccine provides passive immunity and protects her newborn also

Antisepsis

Removing pathogens from living tissue

reflection

Repeating a client's words when the midwife is gathering information for her health history is known as: Answer feedback empathetic response confrontation reflection

-allowing women in normal labor to eat and drink at will throughout the course of their labor

Research supports which of the following approaches to oral intake during labor? Answer -allowing women to drink only water and ice chips during labor -allowing women in normal labor to have clear liquid but no solid food -allowing women in normal labor to eat and drink during early stage of labor but limiting oral intake to liquids as labor progresses -allowing women in normal labor to eat and drink at will throughout the course of their labor

fever

Respiratory distress in a newborn is evidenced by all the following, except: Answer tachypnea grunting retractions fever

insuction and grunting

Respiratory distress in a newborn is evidenced by: Answer -respirations of 50 -floppy muscle tone -insuction and grunting -acrocyanosis

a missed abortion

Ruby is a 30 year old G3P 1011 at 14 weeks gestational age by an ultrasound that she had performed at 8 weeks in her doctor's office. She has now switched to a midwife and is being seen for her first prenatal appointment. She reports that everything has been normal with her pregnancy except for some vaginal bleeding and lower abdominal pain about a month ago that resolved spontaneously. The midwife cannot locate the fetal heart tones with a doppler and finds that the uterine size is smaller than dates. The midwife would suspect which of the following: Answer a threatened abortion an inevitable abortion an incomplete abortion a missed abortion

you can shower when you are ready but I will stay nearby

Sarah is a married, 20 year old gravida 1 para 1001 who just had a beautiful delivery in a birthing center. She lost 350 cc with the birth, and has stable vitals postpartum. Three hours after delivery, she says she wants to take a shower. The midwife's best response would be: Answer "You require bed rest for 4 hours after delivery to minimize blood loss" "You can shower when you are ready, but I will stay nearby" "Showering or bathing may cause you to get dizzy, so it is not recommended" "You are not allowed to shower for 12 hours so you don't get chilled"

3rd degree

The laceration which includes the anal sphincter but does not completely split the sphincter is called a: Answer 1st degree 2nd degree 3rd degree anal rupture

-more easily diagnosed and treated in men than women

Sexually transmitted diseases are: Answer -always treatable with antibiotics -more easily diagnosed and treated in men than women -more easily diagnosed and treated in women than men -easily caught from public restroom surfaces like toilet seats

thirst

Shock patients will often complain of: Answer headaches thirst feeling hot itchiness

effacement

Shortening and thinning of the cervix

Prolonged transition stage

Signs and symptoms of a patient with precipitous labor may include all the following, except: -contraction every 2 minutes, lasting 90 seconds -rapid progressive cervical dilation -prolonged transition stage -discomfort that seems out of proportion to the dilation

cord lengthens

Which of the following is a sign of placental separation? Answer -contractions every 2 minutes -cord is pulsing -cord lengthens -the mother wants to rest

transition

Six hours into the labor, Kate's contractions are occurring every 2 minutes and lasting 80 seconds. She is diaphoretic, restless, irritable, and moaning that she "can't take it anymore". Kate is in labor with her 5th baby. According to these behaviors, what stage or phase of labor is she most likely in? Answer latent phase second stage third stage transition

low birth weight infant

Smoking in pregnancy is associated with an increased incidence of: Answer diabetes maternal anxiety low birth weight infant macrosomia

--Goodell's sign

Softening of the cervix is a sign of pregnancy called: Answer Hegar's sign Chadwick's sign Goodell's sign Pisasck's sign

full bladder

Something that may keep the uterus from contracting is: Answer -a full bladder -not eating during labor -tachycardia -high blood pressure

-the degree of descent of the presenting part

Station refers to what, in relation to the ischial spines? Answer -the degree of descent of the presenting part -the degree of crowning of the head -the degree of engagement of the head -the degree of palpation of the presenting part

Cephalohematoma

Swelling caused by bleeding between the osteum and periosteum of the skull. This swelling does not cross suture lines.

21-22

THe heart begins to beat on day - 9-10 - 21-22 - 28-29 - 32-33

above 160

Tachycardia in a fetus is defined as a heart rate: Answer above 140 above 160 above 190 below 120

fetal infection

Tachycardia in the fetus may indicate what serious condition? Answer -fetal infection -cord prolapse -uterine abruption -fetal anomaly

this is a normal reaction; it allows her to process the reality of her natural birth

Tammy, a gravida 3, para 3003 has had an uncomplicated water birth at home. It was her first natural birth after two previous epidurals in the hospital with her first 2 children. At the 24 hour visit, Tammy's husband tells the midwife in private that he is worried because she has been talking constantly about her recent delivery experience. The midwife's best response would be: Answer "Water birth is special; all women talk this way" "Do you feel uncomfortable talking about the delivery?" "This is a normal reaction; it allows her to process the reality of her natural birth" "Yes, it is a clear sign she is obsessed; we will watch her closely for signs of depression"

high levels of HCG

Tara is a 27 year old gravida 3 para 2 who has missed two menstrual periods and is making her initial visit to the midwife. The midwife gave her a pregnancy test and it was positive. Tara reports nausea and occasional vomiting in the mornings. Which physiologic change of pregnancy is probably causing Shelley's nausea and vomiting? - Altered stomach position -increased gastric motility - reflux of acidic secretions in to the lower esophagus - high levels of HCG

Acrocyanosis

Temporary cyanotic condition, usually in newborns resulting in a bluish color around the lips, hands and fingernails, feet and toenails. May last for a few hours and disappear with warming.

TPAL

Term, preterm , abortion, living children

the craving for non-food, non-nutritive substances such as ice, clay, dirt, starch or plaster

The definition of pica is: -the craving for non-food, non-nutritive substances such as ice, clay, dirt, starch or plaster -the vitamin deficiency that leads to excessive itching -a cultural definition of suppressed appetite caused by pregnancy nausea -the body's reaction to vitamin C deficiency that causes sores all over the body

-sensitization of the Rh-negative mother with Rh+positive antibodies, indicating the potential for increased risk to the fetus

Testing an Rh-negative mother for antibodies determines: Answer -the presence of immunoglobins in the blood of the mother who is Rh-negative -sensitization of the Rh-negative mother with Rh+positive antibodies, indicating the potential for increased risk to the fetus -the ability of the fetus to withstand the increased stress of maternal antibodies in an immunocompromised state -the presence of Rh-negative antibodies in an Rh-negative fetus

cytomegalovirus

The "C" in the TORCH group of diseases refers to: Answer calcium pyrophosphate arthropathy cytomegalovirus carbon pneumoconiosis congenital diaphragmatic hernia

-the curve formed by the sacrum, coccyx, and pubic bones

The Curve of Carus is: Answer -the curve formed by the sacrum, coccyx, and pubic bones -the curve of the fetal neck when the occiput is fully flexed -the curve formed by the sacrum and the ischial spines -the curve formed by the pubic arch

on her back with feet flat

The Dorsal position is when the mother: -is on her back with her legs in stirrups -is on the back with feet flat -is in a semi-sitting position -is in a standing squat

-lie is longitudinal and fetal occiput is directed toward the left anterior portion of the maternal pelvis

The LOA position means that the: Answer -lie is longitudinal and the fetal occiput is directed toward the left posterior portion of the maternal pelvis -lie is transverse and fetal mentum is directed toward the left posterior portion of the maternal pelvis -lie is longitudinal and fetal occiput is directed toward the left anterior portion of the maternal pelvis -lie is oblique and fetal anterior fontanel is directed toward the left posterior portion of the maternal pelvis

L/S ratio

The amount of Lexithin and sphingomyelin found in the amniotic fluid > 2:1 = Mature lungs

-hemoglobin measures the oxygen carrying capacity of the red blood cell; hematocrit measures the volume of red blood cells in a packed volume of blood

The best way to describe the difference between hemoglobin and hematocrit is: Answer -hemoglobin measures the oxygen carrying capacity of the red blood cell; hematocrit measures the volume of red blood cells in a packed volume of blood -there is no difference; they both measure the risk for anemia -hematocrit measures the oxygen carrying capacity of the blood; hemoglobin is the percentage of red blood cells in a given volume of plasma -the hemoglobin represents the same value as the hematocrit, with the hematocrit expressed as three times the value of the hemoglobin

-acrocyanosis

The blue coloration of a newborn's hands and feet is called: -vernix caseosa -acrocyanosis -harlequin color -erythema neonatorum

-directly encircles the amnion layer of the amniotic sac

The chorion layer of the amniotic sac: Answer -directly encircles the fetus -directly encircles the amniotic fluid -directly encircles the amnion layer of the amniotic sac -directly encircles the uterus

-hard, warm nodular areas in the outer quadrant of the breast

The clinical manifestations of mastitis include: -hard, warm nodular areas in the outer quadrant of the breast -cessation or diminishing of milk production -marked engorgement and pain -soft, pliable breasts with a bluish tinged line running up the breast

-to establish a diagnosis of normality and to determine who needs specific care, consultation, or referral

The best description of risk assessment is: Answer -to establish a diagnosis of normality and to determine who needs specific care, consultation, or referral -to determine which clients need support when deciding what level of risk is appropriate for them -a specific laboratory test that will determine level of risk for a particular condition -a baseline for determining if a specific client can be moved from one risk level to another

-have the mother hold them next to her skin like a baby kangaroo

The best way to care for premature or low birth weight babies is to: Answer -try to touch them as little as possible so as not to over stimulate their immature nervous system -have the mother hold them next to her skin like a baby kangaroo -give them formula because they are too weak to suck -keep them in the hospital as long as possible to avoid infection from the home

secretion of hydrogen ions

The kidneys help to regulate the pH of body fluids by controlling the - conversion of acids to bases - secretion of hydrogen ions - filtration of buffers - reabsorption of CO2

is thoroughly pink

The color of a baby is considered a 2 if at the time of the Apgar the baby: Answer -has bluish hands and feet -is whitish and pale in the trunk area -is thoroughly pink -is purplish blue only around the head

a diamond

The anterior fontanel is roughly the shape of: Answer a rhomboid a diamond a triangle an oval

placental separation

The anterior fontanel is roughly the shape of: Answer a rhomboid a diamond a triangle an oval

sagittal suture

The anterior posterior suture is called the: Answer sagittal suture lambdoidal suture coronal suture frontal suture

coronal suture

The anterior transverse suture is called the: Answer sagittal suture lambdoidal suture coronal suture frontal suture

120-140bpm

The average heart beat range for a full-term, quiet and awake newborn is 80-100 beats per minute 100-120 beats per minute 120-140 beats per minute 150-180 beats per minute

120- 160 bpm and irregular

The baseline FHT for a fetus which is developing normally usually is: Answer 100 to 120 beats/minute and irregular 120 to 160 beats/minute and irregular 120 to 160 beats/minute and regular 140 to 180 beats/minute and regular

heart

The basic shape of the android pelvis is: Answer round transverse oval anterior/posterior oval heart

-anterior/posterior oval

The basic shape of the anthropoid pelvis is: Answer -round -transverse oval -anterior/posterior oval -heart

inominate bones

The coxae, or hip bones are commonly referred to by another name in many medical midwifery text books. What is the common term for these two bones? - tuberosities - inominate bones - sacral bones - pelves

-a disinfectant is used on lifeless objects only, while an antiseptic is used on the skin's surface

The difference between a disinfectant and an antiseptic is that: Answer -a disinfectant is used on lifeless objects only, while an antiseptic is used on the skin's surface -a disinfectant kills bacterial spores, while an antiseptic does not -a disinfectant does not achieve sterilization and an antiseptic does -disinfectants are chemical substances, while antiseptics are not

2-8 weeks

The embryonic stage is defined as: Answer 3-21 weeks 16-20 weeks 1-4 weeks 2-8 weeks

ATP

The energy currency of the cell - ATP - NAD - ADP - Ca2+

midline

The episiotomy cut that is said to heal the fastest is: Answer -midline -medio-lateral -several snips instead of one -large cut deep

--estrogen

The essential female hormone responsible for reproduction is: Answer testosterone estrogen prolactin prostaglandin

testosterone

The essential male hormone responsible for reproduction is: testosterone estrogen progesterone prostaglandins

-non-stress test, fetal movements, fetal breathing movements, fetal heart tones, amniotic fluid volume, placental grading

The evaluation of a biophysical profile includes which of the following: Answer -fetal movement, fetal urinary bladder volume, blood flow -placental measurements, fetal breathing patterns, blood viscosity, presence of fetal heart accelerations -placental grading, genetic testing of amniotic fluid, fetal reactivity, fluid volume -non-stress test, fetal movements, fetal breathing movements, fetal heart tones, amniotic fluid volume, placental grading

in the groin

The femoral pulse is located: Answer on the wrist in the axilla in the groin on the neck

fetal well being

The fetal heart tones are staying between 140-150 BPM during labor, which indicates: Answer maternal hypoxia fetal lung maturity fetal movement fetal well-being

18-20 weeks

The fetal heartbeat can first be heard with a fetoscope between (from LMP): Answer 2-5 weeks 6-8 weeks 10-14 weeks 18-20 weeks

9-40 weeks

The fetal stage is defined as: Answer 3-21 weeks 16-20 weeks 9-40 weeks 2-8 weeks

9 weeks

The fetal stage of development starts at what week of gestation (from conception)? Answer 6 weeks 9 weeks 16 weeks 20 weeks

umbilical vein

The fetus receives oxygenated blood through the: Answer ductus arteriosus foramen ovale mother's arteries umbilical vein

chills with an elevated temp

The first indication of puerperal infection is usually: -excessive bleeding -chills with an elevated temperature -vomiting and nausea -severe abdominal pain

-onset of true contractions to full dilation

The first stage of labor is defined as: Answer -the start of contractions to the end of them -onset of true contractions to the birth of the baby -onset of true contractions to full dilation -onset of a contraction to transition

risk assessment and patient teaching

The first step in preventing preterm labor is: Answer -remembering the causative factors -risk assessment and patient teaching -early management with tocolytic agents cervical -cerclage at 28 weeks gestation

-a persistent inconsolable cry is common in infants before the milk comes in

The following are generally true about the character of an infant's cry, except: Answer -a persistent weak cry may mean an infant is ill -a persistent high pitched cry may signify intracranial injuries -a persistent inconsolable cry is common in infants before the milk comes in -it is normal for the crying to stop when the baby is picked up

-the anterior fontanel is triangular and bounded by the frontal bones and the parietal bones

The following are true about the fetal skull except: Answer -the anterior fontanel is triangular and bounded by the frontal bones and the parietal bones -the sagittal suture is the antero-posterior division between the parietal bones -the biparietal diameter is the largest transverse diameter of the head -the bitemporal is the greatest distance between the two temporal structures

right atrium to left atrium

The foramen ovale is an opening that allows fetal blood to pass from the - right atrium to left atrium - left atrium to right atrium - right ventricle to left ventricle - left ventricle to right ventricle

provide a storage mechanism for glucose that will be used later to replenish amniotic fluid

The function of the placenta is all of the following, except: -provide somewhat of a barrier against certain infections -provide a storage mechanism for glucose that will be used later to replenish amniotic fluid -provide nutrients and hormones needed by the fetus -act as an organ of exchange and transfer

provide nutrients

The function of the placenta is to: Answer -prevent infection -mix the fetal and maternal blood -provide nutrients -allow for fetal movement

NEPHRON

The functional unit of the kidney is - the nephron - the neuron - the renal corpuscle - the renal pyramid

WIC

The government program which provides food to pregnant and nursing women and their children in the USA is called: Answer FDA HIPPA HUD WIC

most conducive to a vaginal birth

The gynecoid pelvis is: Answer the least frequently occurring pelvis most conducive to a vaginal birth the male pelvis a flat pelvis

4 weeks

The heart begins beating in the embryo during which week (from conception)? Answer 2 weeks 6 weeks 1 week 4 weeks

---progesterone

The increased activity of the endometrial glands during the luteal phase of the female reproductive cycle is stimulated by: Answer progesterone estrogen glycogen prolactin

-progesterone

The increased activity of the endometrial glands during the luteal phase of the female reproductive cycle is stimulated by: - progesterone - estrogen -oxytocin -prolactin

prominent

The ischial spines of the android pelvis are: Answer wider apart than any pelvic type prominent blunt not easily felt

-accept the fact that her attention has been temporarily withdrawn from him and centered on herself

The midwife can best help the father cope with his wife's pregnancy by suggesting that he: Answer -supervise her dietary intake and daily activities to make sure she adheres to instructions -accept the fact that her attention has been temporarily withdrawn from him and centered on herself -protect her from all responsibilities and problems throughout the pregnancy -keep her busy and occupied so that she will not become too introspective

-helping her give the baby a bath

The midwife could best help a new mother learn parenting skills by: Answer -bathing the baby for her -demonstrating with a doll how to bathe a baby -helping her give the baby a bath -sending her home with a video on newborn care

green leafy vegetables

The midwife diagnoses slight megaloblastic anemia from folic acid deficiency during a routine prenatal checkup at 25 weeks gestation. The midwife would recommend that the woman eat more: Answer -fish, cheese, and eggs -green, leafy vegetables -milk and milk products -citrus fruits

placental abruption

The midwife has been at Mary's house for one hour. Upon arrival, she was 2 cm, BP 130/84, and FHT was 152. She is experiencing contractions as very painful and flinches noticeably when her abdomen is touched. The midwife has just taken FHT again one hour later and find them to be 100. BP is 90/60. She would most suspect: Answer cord prolapse infection placental abruption placenta previa

-breastfeeding every 2-3 hours or on demand, whichever comes first

The midwife has been helping answer the questions of a woman who has just given birth for the first time. Which behavior shows that the new mother understands correct teaching on breastfeeding? -breastfeeding every 4 hours -allowing the baby to breastfeed for only 3-5 minutes from each breast -breastfeeding every 2-3 hours or on demand, whichever comes first -supplementing her breastfeeding with glucose water in the first three days

oxytocin

The midwife has just checked the fundus of a woman who gave birth 45 minutes ago and finds it is firmly contracted. The mother is happily nursing her new baby. What hormone is most likely being released that will assist uterine involution? Answer oxytocin adrenaline progesterone estrogen

-negative indirect Coombs test result

The midwife is caring for a pregnant woman whose blood is Rh-negative. The midwife would discuss administration of RhoGAM at the 28th week of gestation if the woman had a: Answer -negative direct Coombs test result -negative indirect Coombs test result -positive direct Coombs test result -positive indirect Coombs test result

obtain a blood specimen to test for IgG and IgM antibody titers and consult with a physician

The midwife is caring for a young woman who was never immunized as a child and does not believe in routine immunization. She comes in tears to report that she has been exposed to rubella at the day care where she works. She is currently 27 weeks pregnant. The lab results from her initial prenatal visit indicate that she is rubella non-immune. Which of the following is the most appropriate action for the midwife to take next in the management of this client? Answer -tell her to watch to see if she develops clinical signs of disease and if they appear, tell her to take a week off of work -tell her she needs to be immunized against rubella within 96 hours of exposure -order an ultrasound to look for signs of fetal rubella infection -obtain a blood specimen to test for IgG and IgM antibody titers and consult with a physician

late

The midwife is listening during a contraction and hears the fetal heart rate drop following the acme of the contraction. What kind of decelerations are occurring? Answer early variable late unknown

-round, firm object low in the pelvis, small parts on the mother's left side, and a soft rounded shape in the fundus

The midwife is performing Leopold's maneuvers and determines that the fetus is ROA. Which of the following did she find? -round, firm object low in the pelvis, small parts on the mother's right side, and a soft rounded shape in the fundus -round, firm object low in the pelvis, small parts on the mother's left side, and a soft rounded shape in the fundus -soft rounded shape on the mother's right side, firm rounded shape on the mother's left side, and small parts at the level of the umbilicus -soft rounded shape in the lower pelvis, small parts on the mother's right side, and a firm, round object in the fundus

fetal head in ROA position

The midwife is performing Leopold's maneuvers during a prenatal exam on a woman at 34 weeks. She feels kicks as she is palpating on the mother's left side, and discovers that the fetal back is on the mother's right side. This is indicative of: Answer fetal head in LOA position fetal head in LOP position fetal head in ROA position fetal head in ROP position

uteroplacental insufficiency

The midwife listens during and after a few contractions and notes that a deceleration occurs about 30 seconds after each contraction begins, returning to baseline after the contraction is over. This type of deceleration is thought to be caused by: Answer fetal head compression umbilical cord compression uteroplacental insufficiency cardiac anomalies

-apply manual pressure to the presenting part and have the mother assume a knee-chest position

The midwife notes a cord protruding from a laboring woman's vagina. The first action would be to: -call an ambulance and then call the physician -start administering oxygen by mask and assess the mother's vital signs -place a clean towel on the cord and wet it with sterile saline solution -apply manual pressure to the presenting part and have the mother assume a knee-chest position

-such behavior is not unusual; it is a way an expectant father can explore his feelings

The midwife notices a client's husband exhibiting some of the symptoms associated with couvade syndrome during a conversation about pregnancy discomforts. The midwife should understand that: Answer -such symptoms usually are physiologically based -such behavior is a father's attempt at bonding with his unborn child -such behavior is not unusual; it is a way an expectant father can explore his feelings -such behavior must be redirected to promote a healthy father image

readjust her position and listen again

The midwife notices that while listening during one contraction she cannot hear the fetal heart tone with her fetoscope. She should immediately: Answer -administer oxygen -turn the woman to her left side -readjust her position and listen again -record on the chart that she is unable to hear the FHT at this time

-aspirate the oral and nasal pharynx with a bulb syringe

The midwife sees a two hour old newborn gagging and turning blue. The first thing the midwife should do is: -try to get him breastfeeding -aspirate the oral and nasal pharynx with a bulb syringe -give oxygen by positive pressure ventilation -raise the infant's head and stimulate crying by slapping the soles of the feet

-respiratory depression and loss of deep tendon reflexes

The midwife should be aware of clinical manifestations of magnesium sulfate toxicity, which include: Answer -headache, blurred vision, and 3+ proteinuria -respiratory depression and loss of deep tendon reflexes -flushing and sweating -ringing in the ears and a metallic taste in the mouth

enhance fetal descent and prevent pp uterine atony

The midwife should encourage frequent voiding during labor, primarily to: Answer -enhance fetal descent and prevent postpartum uterine atony -prevent UTI and the need for antibiotics -strengthen the vaginal and perineal muscles -assess urine specimens for albumin and ketones

over the shoulder of the fetus

The midwife should place the fetoscope or doppler: Answer -over the fundus -over the shoulder of the fetus -over the mother's pubic bone -over the mother's umbilicus

it is less expensive and less complicated

The midwife should tell a couple seeking infertility testing that the husband is tested first for infertility because: Answer -it is less expensive and less complicated -the incidence of infertility is higher in men -male infertility is not responsive to therapy -the cause of male infertility takes longer to identify

inform the mother that the rate is normal

The midwife takes fetal heart tones and finds they are 158 bpm. Which of the following actions is appropriate? -inform the mother that the rate is normal -reassess the FHT in 5 minutes because it is too high -change the mother's position because the FHT are too high -tell the mother she is having a girl because the heart rate is fast

-identify the fetus as a separate human being

The midwife would most expect a woman entering the second trimester to experience which of the following emotions: Answer -feel ambivalent about the pregnancy -identify the fetus as a separate human being -anticipate her child's first year of life -realistically prepare for childbirth

-to ensure adequate protein and caloric intake in order to establish good uteroplacental exchange

The most appropriate nutritional recommendation when confronted with a woman at risk for pre-eclampsia is: Answer -to decrease protein and calories to prevent excessive weight gain, which can lead to LGA and cesarean section -to restrict sodium and treat with diuretics -to ensure adequate protein and caloric intake in order to establish good uteroplacental exchange -to discourage excessive sugar and fat intake in order to decrease stress on the liver

-abnormalities in the embryo

The most common cause of early spontaneous abortion is: Answer -systemic infection in the mother -abnormalities in the embryo -placental abnormalities -physical trauma

fallopian tubes

The most common implantation site associated with ectopic pregnancies is: Answer abdomen cervix fallopian tubes ovaries

hypovolemic

The most common type of shock a midwife will encounter is: Answer -cardiogenic -septic -hypovolemic -anaphylactic

CONVULSIONS

The most diagnostic feature of eclampsia is: Answer hypertension edema visual disturbances convulsions

occiput

The most posterior portion of the cranium.

endometrium

The non-pregnant uterus is lined by the: Answer endometrium myometrium decidua vera decidua capsularis

-abdominal with synchronous chest movements

The normal breathing pattern for a full-term baby is predominately: -chest breathing with nasal flaring -abdominal with synchronous chest movements -diaphragmatic with chest lag -shallow and irregular respirations

placenta

The organ responsible for fetal respiration prior to delivery is the: Answer placenta lungs amniotic fluid fetal skin

presentation

The part of the fetus which lies at the pelvic brim or in the lower pole of the uterus tells the: Answer presentation attitude lie posture

the resting phase

The period during labor between complete dilation and the onset of active effective pushing is called: Answer active pushing the resting phase crowning latent phase

weeks 8-28

The period of time when the baby grows faster than at any other time during life is - weeks 2-8 - weeks 8-12 - weeks 3-9 - weeks 8-28

prevention of uterine motility during pregnancy

The physiologic effects of estrogen do not include: Answer -development of secondary sex characteristics -prevention of uterine motility during pregnancy -myometrial thickening -maturation of ovarian follicles

muscular peristalsis of the fallopian tube

The physiologic effects of progesterone do not include: Answer muscular -peristalsis of the fallopian tubes -increased basal metabolism -placental growth -development of acini cells in the breasts

chorionic vill and decidua basalis

The placenta forms from the: Answer -chorionic villi and decidua vera -chorionic villi and decidua capsularis -decidua basalis and decidua vera -chorionic villi and decidua basalis

hypothalamus

The posterior pituitary lobe is a continuation of the: Answer hypothalamus thyroid adrenals placenta

lambdoidal suture

The posterior transverse suture is called the: Answer sagittal suture lambdoidal suture coronal frontal

asian

The prevalence of Rh-negative women in a given geographic area varies by race. Which race is least likely to be Rh-negative? Answer Caucasian African American Asian Hispanic

-maintain the corpus luteum during the first trimester

The primary function of human chorionic gonadotropin (hCG) is to: -confirm the diagnosis of pregnancy -maintain the corpus luteum during the first trimester -nourish the fetus -stabilize the uterus during pregnancy

-deafness, heart defects, cataracts, and cognitive disabilities

The risks to a fetus of a woman who is not immune to rubella include: Answer -gastrointestinal malformations -reproductive abnormalities, hypoglycemia, and learning disabilities -deafness, heart defects, cataracts, and cognitive disabilities -lung tissue immaturity, limb malformation, failure to thrive

synclitism

The sagittal suture of the fetal head is felt midway between the symphysis pubis and the sacral promontory. This would indicate: Answer -synclitism -asynclitism -deep transverse arrest -engagement

full dilation to birth of the baby

The second stage of labor is defined as: Answer -the start of active contractions to the end of them -full dilation to the birth of the baby -onset of true contractions to full dilation -onset of transition to the birth of the baby

-ultrasound image of the baby, hearing fetal heart tones

The signs that constitute positive signs of pregnancy are: Answer -enlarging uterus, Braxton-Hicks contractions, fetal movement -ultrasound image of the baby, hearing fetal heart tones -Hegar's sign, Chadwick's sign, hCG positive -nausea, breast tenderness, urinary frequency

-elevated blood pressure, generalized edema, proteinuria

The three classic signs of pre-eclampsia are: -elevated blood pressure, generalized edema, proteinuria -glucosuria, dizziness, headaches -elevated blood pressure, bleeding, anemia -hyperreflexia, pitting edema, infection

genes

The transmission of potential hereditary characteristics is the function of the: Answer amnion gametes sex hormones genes

ovoid

The types of pelvis may be classified into any of the following, except: Answer gynecoid ovoid android anthropoid

-three vessels: one artery and two veins

The umbilical cord has: Answer -three vessels: one artery and two veins -two vessels; one artery and one vein -Wharton's jelly, connective tissue, and bone marrow

--decidua

The uterine endometrium during pregnancy is called the: Answer decidua lining myometrium chorion

the lower uterine segment

The uterine isthmus is essential during pregnancy in the formation of: Answer the external os the lower uterine segment Bandl's ring the placenta

external os

The vaginal portion of the cervix that dilates during labor is called the: Answer external os internal os anterior lip posterior lip

HPV

The virus involved in genital warts is: Answer genital herpes herpangina human papilloma human immunodeficiency

a fern test

Theresa, a gravida 1, is admitted to the birth center in labor. She complains of abdominal pain that seems to "come and go" and states that her water may have broken on the way to the birth center. Which procedure would best determine if Theresa has SROM? Answer a CBC a fern test urinalysis a vaginal examination

-narrow pubic arch

Which of the following is associated with the android pelvis? Answer -oval shaped inlet -narrow pubic arch -excellent labor patterns -a well curved sacrum

IgG

This class of antibody is the only one that can cross the placenta to provide immunity to the fetus - IgM - IgG - IgE

IgM

This is the first immunoglobulin released during an immune response - IgM -IgG - IgE

lactation amenorrhea method

Through exclusive breastfeeding, the woman is able to suppress ovulation

candida albicans

Thrush in the newborn is directly related to contact in the birth canal with which of the following organisms? -candida albicans -neisseria gonorrhoea -treponema pallidum -staphylococcus aureus

Protein

Which nutrient helps with the formation of hormones, enzymes, and antibodies, is a source of heat /energy, and maintains your acid/alkili balance? - protein - folic acid -vitamin c - carbs

-extend her legs and dorsiflex each foot several times before bedtime

To best help prevent leg cramps from disturbing her sleep, the midwife would recommend that a pregnant woman should: Answer -extend her legs and dorsiflex each foot several times before bedtime -elevate and support her legs on pillows while sleeping -massage her calves vigorously with lotion before bedtime -drink a cup of green tea before bedtime

-rotate the site she takes the temperature from; i.e., from mouth to armpit, etc

To properly take an accurate basal body temperature to determine ovulation, a woman should do all of the following, except: -use a basal thermometer -take temperature after 5-6 hours of uninterrupted sleep -rotate the site she takes the temperature from; i.e., from mouth to armpit, etc -take it before any activity, even going to the bathroom

-avoid eating inadequately cooked meat

To protect her unborn child from toxoplasmosis, a pregnant woman should: -avoid people known to have German Measles -avoid eating inadequately cooked meat -avoid sexual relations with known carriers -be vaccinated against it early in her pregnancy

IV Fluids

To replace blood volume lost, a woman in hypovolemic shock should be given: Answer -hypotensive medications -water to drink -food with high water content to eat -IV fluids

-during the birth process

Transmission of the herpes virus can occur to the newborn: Answer -during the birth process -from contaminated formula -in airborne droplets from the mother -from infected breasts

-almond size and somewhat tender

Upon palpating the ovaries during a pelvic exam, the midwife would expect normal, healthy ovaries to be: Answer -of different sizes and irregular -walnut size and very tender -almond size and somewhat tender -peanut size and not tender

vitamin D

Which nutrient improves absorption and use of calcium, maintains the stability of your nervous system, and helps ensure normal heart action? - Vitamin E - Vitamin K - Vitamin D - Vitamin C

false

True or false Taste receptors may be enhanced in pregnancy leading to cravings for highly seasoned foods

true

True/false The lungs move up and tidal volume increases by 40% in pregnancy

-massage the uterus firmly with one hand while supporting it with other hand

Twelve hours postpartum, the midwife checks the mother's fundus and finds it enlarged and boggy. What would the midwife do first? -take her vital signs to rule out hypertension so she can give methergine -check the consistency of the fundus at 15 minute intervals and chart this information -administer an oxytocic drug -massage the uterus firmly with one hand while supporting it with other hand

estrogen and progesterone

Two hormones the placenta produces are: -oxytocin and estrogen -progesterone and oxytocin -estrogen and progesterone -prolactin and estrogen

60-80bpm

What is the normal range of pulse in an adult? Answer 10-15 bpm 15-20 bpm 60-80 bpm 80-100 bpm

hormone changes

Urinary frequency in early pregnancy is most often a result of: Answer -increased thirst leading to increased fluid intake -a urinary tract infection -pressure from the enlarging uterus on the bladder -hormone changes

February 19

Using Naegel's formula, the EDD for a woman who reports her LMP as May 12 would be: Answer December 9 February 19 February 9 August 5

January 20

Using Naegel's rule, the EDD for a woman who reports her LMP as April 13 would be: Answer December 20 January 20 January 2 February 1

2 term, 2 preterm, 0 abortions, 3 living children, 1 multiple birth

Using the 5 digit Para system, what does 2 2 0 3 1 signify? Answer -2 term, 2 abortions, 0 preterm, 3 living children, 1 miscarriage -2 term, 2 preterm, 0 abortions, 3 living children, 1 multiple birth -2 term, 2 preterm, 0 abortions, 3 living children, 1 miscarriage -2 preterm, 2 abortions, 0 term, 3 living children, 1 multiple birth

-autolysis of protein material within the uterine wall

Uterine involution occurs as a result of: -a decrease in the number of myometrial cells -necrosis of the hypertrophic myometrial cells -autolysis of protein material within the uterine wall -necrotic degeneration of the placental site

cord compression

Variable decelerations are thought to be caused by: Answer -head compression -cord compression -uteroplacental insuffiency -maternal pushing efforts

O type blood

Which blood type is the universal Donor? - A - B - AB - O

-"If it feels good, go ahead and push, if it does not feel good, don't push yet"

Veronica is a G5P4 who is in hard labor at home. She was 9 cm an hour ago, and now she has involuntary grunting with the last contraction and asks the midwife if she can push. The midwife tells her: Answer -"You are not allowed to push until I check your cervix" -"Breathe and pant for the next few minutes" -"If it feels good, go ahead and push, if it does not feel good, don't push yet" -"Push as hard as you can with the next contraction and see if you can move the baby down"

-newborns lack the intestinal bacteria with which to synthesize vitamin K

Vitamin K is administered in the immediate neonate period because: -a newborn's liver is incapable of producing sufficient vitamin K to deal with transient neonatal coagulation problems -hemolysis of the fetal red blood cells increase coagulation problems -newborns are susceptible to changes in temperature, which increase jaundice -newborns lack the intestinal bacteria with which to synthesize vitamin K

obstetric history

Wanda is a 22 year old gravida 3, para 0201 who calls the midwife during her 32nd week of gestation reporting contractions every 10 minutes, lasting for 45 seconds for the past 4 hours. She says the contractions are increasing in intensity, not dissipating with rest. The midwife visits and finds her dilated to 2 cm, 50% effaced and 0 station, membranes intact. Wanda is at increased risk for preterm labor because of her: Answer age cervical dilation anxiety obstetric history

thrush

What do white spots on the infant's tongue and gums that cannot be easily removed indicate? Answer -strep throat -milk residue -thrush -milk intolerance

-physical exam of head, eyes, ears, nose and throat

What elements of a physical exam are included in HEENT? Answer -physical exam of hair, eyes, ears, nose and throat -physical exam of head, eyes, ears, nose and torso -physical exam of head, eyes, ears, nose and throat -physical exam of hair, eyes, ears, nose and thyroid

b12

What essential vitamin is a strict vegan mother most likely to be deficient in? Answer Vitamin C Vitamin B12 Vitamin D Vitamin A

separation of a previous c section scar

What is the most common cause of uterine rupture? Answer -separation of a previous cesarean section scar -congenital anomalies of the uterus -labor stimulation/augmentation with oxytocin or prostaglandins -external cephalic version

-hypoxic-ischemic encephalopathy

What is the most frequent reason for seizures in the neonatal period? Answer -side effects of maternal medication/drug use -hypoglycemia -hyperkalemia -hypoxic-ischemic encephalopathy

family history

What is the most important known risk factor for breast cancer: Answer genetic alterations diet family history age at first pregnancy

foramen ovale

What is the name of the temporary opening between the atria in the fetal heart which allows blood to pass since it doesnt need to pass through the lungs -ductus arteriosis -formen ovale -umbilical vein -left atria

12-20

What is the normal range for respirations in an adult? Answer 10-15 12-20 60-80 80-90

-keep her hands off the breech until it is born to the umbilicus

When delivering a breech, as the body begins to deliver, the midwife should: Answer -wrap a towel around the baby and start pulling at the hips -make the mother lie down -keep her hands off the breech until it is born to the umbilicus -lift the body and rotate anteriorly then posteriorly to promote head flexion

8-12 weeks

When does HCG peak in pregnancy - 1-2 weeks - 4-8 weeks - 8-12 weeks -12-20 weeks

-counseling women about their sexual choices and preferences

When doing a physical exam on a woman, it does not include: Answer -a relevant history along with a physical and pelvic examination -laboratory tests related to womens' health care -counseling women about their sexual choices and preferences -addressing concerns about domestic violence, HIV, and substance abuse

abnormalities

When examining the placenta and cord, the midwife is mainly checking for: Answer calcifications abnormalities cysts size

10-12 weeks

When is the first time fetal heart tones are heard by the doppler? Answer 7-10 weeks 12-14 weeks 10-12 weeks 16-18 weeks

during labor and delivery

When is the mother-to-child transmission (MTCT) of the human immunodeficiency virus (HIV) most likely to occur? Answer -in the first 20 weeks of pregnancy -in the third trimester -during labor and delivery -during breastfeeding

inside the external os

When obtaining a Papanicolaou (Pap) smear, the midwife removes cells from the squamocolumnar junction located: Answer -within the cervical canal -inside the internal os -inside the external os -at the cul-de-sac of Douglas

-coccyx

When performing a pelvic exam, one of the bony elements the midwife will identify is located at the end of the sacral curve, and should be slightly moveable. This is the: Answer ischial spine symphysis pubis sacrum coccyx

ischial spines

When performing a pelvic exam, the midwife will feel this on both sides of the pelvis; it could be any of the following: protruding, sharp, non-prominent, or blunt. This is the: Answer ilium ischium sacrum ischial spine

-during a contraction as head crowns

When performing an emergency episiotomy, the midwife must cut: Answer -only after administering a local anesthetic -slowly, with several small snips -between contractions, as head descends -during a contraction as head crowns

--cephalic prominence

When performing the fourth maneuver of Leopold's on a fetus in a cephalic presentation, what will the examiner's fingers strike first? Answer occiput sinciput cephalic prominence mentum

between contractions

When should the midwife assess a laboring woman's blood pressure? Answer -during the increment of a contraction -during the acme of a contraction -during the decrement of a contraction -between contractions

duncan

When the maternal side of the placenta delivers first, it is called: Answer shultz swartz dunblane duncan

suction the mouth first

When the midwife suctions a baby that is unresponsive and not breathing at birth, she should: Answer -suction the nose first -suction using a delee if the baby was breech -suction the mouth first -never suction the baby under any condition

the fallopian tubes

Where does fertilization usually occur? Answer the fallopian tubes the ovaries the uterus the peritoneal cavity

Point of maximal impulse (PMI)

Where heartbeat is best palpable on chest wall; 5th intercostal space, midclavicular line

sacral promontory

Where the first sacral vertebrae bulges into pelvic cavity

megaloblastic Anemia

Which anemia is caused by: severe b12 deficiency, mild folate deficiency, and/or chemotherapy? - microcytic -megaloblastic - normocytic

foul smelling lochia

Which assessment finding is most indicative of postpartum infection? Answer elevated body temperature increased pulse rate a white blood cell count of 25,000 foul smelling lochia

cervical dilation of 2-3 cm

Which assessment finding is not a contraindication for using a tocolytic agent to manage preterm labor? Answer active vaginal bleeding fetal distress cervical dilation of 2-3 cm cervical dilation of 4-5 cm

-a tender, increasingly rigid uterus and increased maternal pulse rate

Which assessment findings are characteristic of abruptio placenta? Answer -a tender, increasingly rigid uterus and increased maternal pulse rate -a soft uterus and maternal hypotension -a tender uterus, a soft abdomen, and decreased maternal pulse rate -a rigid uterus, increased urine output, and a decreased maternal pulse rate

cuddling and inspecting the newborn

Which behaviors should the midwife expect to see when observing a mother and her new baby during the first hour after delivery? Answer -cuddling and inspecting the newborn -sleeping -watching the newborn as it rests beside her -having the father hold the baby

breath holding for no more than 5 seconds while pushing with open glottis

Which breathing pattern should the midwife suggest for use during early second stage? Answer -slow panting -fast panting -breath holding for no more than 5 seconds while pushing with open glottis -breath held pushes while a coach counts to 10, pushing with closed glottis

-increase in blood pressure of 30/15

Which change in the cardiac system is not expected during pregnancy? Answer -increase in cardiac volume -increase in blood pressure of 30/15 -heart displacement to the left -increase of 10 beats/minute in resting heart rate

restoration of diploid number of chromosomes

Which change occurs at fertilization? Answer -meiosis and cell differentiation -gamete division into two cells -establishment of sexual characteristics with further division into the haploid number of cells -restoration of diploid number of chromosomes

surfactant production

Which characteristic normally is not present in a fetus at 20 weeks gestation? Answer surfactant production active movement meconium in the intestinal tract vernix caseosa covering the skin

-amniocentesis

Which diagnostic screening test carries the highest risk of spontaneous pregnancy loss? Answer -ultrasound -CBC with differential -amniocentisis -alpha fetoprotein

the duration of the rest phase between contractions

Which factor would be most helpful in estimating the adequacy of placental perfusion? Answer -the intensity of her contractions -her ability to cope with the pain of labor -the duration of the rest phase between contractions -the effectiveness of her breathing techniques during contractions

umbilical vein--

Which fetal blood vessel carries blood with the highest oxygen content in utero? Answer umbilical vein umbilical artery fetal capillaries pulmonary artery

cheese and yogurt

Which foods should the midwife recommend to increase calcium intake: Answer legumes and enriched cereals tuna and cottage cheese green, leafy vegetables cheese and yogurt

cabbage and bananas

Which foods would provide a pregnant woman with the most vitamin C? Answer cabbage and bananas fish and cottage cheese milk and plain yogurt milk and oatmeal

lean beef, organ meats, and dried fruit

Which foods would supply a pregnant woman with the best sources of iron? Answer fish, chicken, and lean beef broccoli, spinach, and fish eggs, legumes, and dried fruit lean beef, organ meats, and dried fruit

--estrogen--

Which hormone is responsible for the production of the clear, elastic cervical mucus associated with spinnbarkeit and ferning? Answer progesterone prostaglandin estrogen FSH

prolactin

Which hormone stimulates and maintains milk production after childbirth? Answer estrogen progesterone calcitonin prolactin

FSH

Which hormone stimulates maturation of a follicle - FSH -LH -Estrogen - Progesterone

LH

Which hormone stimulates ovulation - FSH -LH - Estrogen - Progesterone

oxytocin--

Which hormone stimulates the let down reflex during breastfeeding? Answer prolactin estrogen progesterone oxytocin

IgG

Which immunoglobulin is most associated with permanent immunity? - IgM - IgG - IgE

extending the leg and dorsiflexing the foot

Which intervention would be most appropriate to help relieve leg cramps during pregnancy? Answer -extending the leg and dorsiflexing the foot -abducting the leg and dorsiflexing the foot -applying pressure to the right knee while plantar-flexing the foot -helping her stand on the leg

-blood vessels between the cranial bone and the periosteal layer rupture, leading to bleeding in the subperiosteal space

Which is most descriptive of a cephalohematoma: -a subcutaneous collection of edematous fluid on the infant's head -blood vessels between the cranial bone and the periosteal layer rupture, leading to bleeding in the subperiosteal space -it typically disappears within 24 hours of birth -traction and rupture of paravaginal veins by the oncoming presenting part

dysuria

Which is not a sign of pre-eclampsia -hypertension -proteinuria -dysuria -pitting edema

-ovulation, fertilization, blastocyst formation

Which is the correct chronological sequence? Answer -ovulation, fertilization, blastocyst formation -implantation, blastocyst formation, ovulation, fertilization -ovulation, fertilization, fetal development, implantation -ovulation, fertilization, embryo formation, implantation

left lateral

Which labor position is preferred for a woman with high blood pressure? Answer left lateral semi-fowler's squatting lithotomy

ADMINISTER RHOGAM

Which measure is essential after an Rh-negative woman undergoes an amniocentesis? Answer -performing a direct coomb's test -performing an indirect coomb's test -administering an antibiotic -administering RhoGAM

a positive ortolani's sign

Which neonatal assessment finding should the midwife report to a physician if she finds this while conducting a newborn exam at one hour after birth? - a positive blink reflex - a positive ortolani's sign - respiratory rate of 70 breaths per minute - a pulse of 170 beats per minute

folic acid

Which nutrient aids in red blood cell formation, helps metabolize protein, and assists in the growth and division of body cells? - fat - vitamin A - protein - folic acid

carbohydrates

Which nutrient gives you energy and aids with digestion? - protein - carbohydrates - fat - vitamin D

Vitamin A

Which nutrient helps the growth and repair of tissue, aids in bone and tooth formation, keeps eyes healthy, destroys carcinogens, and promotes germ killing enzymes? - Protein - b vitamins - vitamin A - vitamin C

Vitamin K

Which nutrient helps with blood coagulation? - Fat -vitamin k - folic acid - vitamin D

Fat

Which nutrient helps with energy, and is a carrier for vitamins A,D,E, and K? - fat -protein - b vitamins - vitamin D

little or no effacement with 1 to 2 centimeter or more dilation

Which of the following best describes the cervix of the average multigravida on the verge of true labor? Answer -50% to 100% effaced with a fingertip to 1 centimeter dilation -50% to 100% effaced with 2 to 3 centimeters dilation -little or no effacement with a fingertip to 1 centimeter dilation -little or no effacement with 1 to 2 centimeter or more dilation

-optimal nutrition is directly linked to optimal health and perinatal outcomes for mother and baby

Which of the following best describes the need for good nutrition in pregnancy? Answer -it allows blood pressure to contract appropriately due to the increased stress on the body in pregnancy -it gives the midwife a chance to discuss the best diet for all pregnant women and to discuss the changes required of her client -optimal nutrition is directly linked to optimal health and perinatal outcomes for mother and baby -it is understood that the typical American diet is adequate for providing all necessary nutrients in pregnancy

prior to conception and through the first 8 weeks

Which of the following best describes the optimal time for folic acid supplementation in order to reduce the risk of neural tube defects? Answer -preconceptually for a year before trying to have a baby -for the first trimester of pregnancy -for 1 to 3 months prior to pregnancy and through the first week of pregnancy -prior to conception and through the first eight weeks

-to explain and schedule appropriate follow-up diagnostic tests

Which of the following best describes the role of the midwife following an abnormal result on a genetic screening test: Answer -to help the woman and her partner cope with the results and help them prepare for the child's special needs -to transfer care of the woman to a physician who specializes in high-risk pregnancies -to explain and schedule appropriate follow-up diagnostic tests -to make an appointment for them with a genetic counselor

Sucking by the newborn

Which of the following best describes the trigger that causes milk ejection from the breast? Answer -fall in progesterone and estrogen levels -increase in human placental lactogen (HPL) -suppression of oxytocin release -sucking by the newborn

-the last day during which the cervical mucus is clear, slippery, and stretchy

Which of the following changes in cervical mucus signals the "peak day" in the menstrual cycle during which ovulation is most likely to occur? Answer -the first day during which the cervical mucus is thick, sticky, and tacky -the first day in which the cervical mucus is clear, slippery, and stretchy -the last day during which the cervical mucus is thick, sticky, and tacky -the last day during which the cervical mucus is clear, slippery, and stretchy

1st - 1 to 12 weeks, 2nd - 13 to 27 weeks, 3rd - 28 to 40 weeks

Which of the following correctly divides pregnancy into three trimesters by weeks, according to LMP? Answer 1st - 1 to 12 weeks, 2nd - 13 to 24 weeks, 3rd - 25 to 40 weeks 1st - 1 to 13 weeks, 2nd - 14 to 27 weeks, 3rd - 27 to 40 weeks 1st - 1 to 12 weeks, 2nd - 13 to 27 weeks, 3rd - 28 to 40 weeks 1st - 1 to 20 weeks, 2nd - 21 to 30 weeks, 3rd - 31 to 40 weeks

ruptured tubal pregnancy

Which of the following dangerous complications of pregnancy would be characterized by shoulder pain? -gallbladder infection -peptic ulcer -ruptured tubal pregnancy -cancer of the cervix

-contractions that last for 30 seconds with a 2 1/2 minute rest in between

Which of the following describes contractions with a frequency of 3 minutes? -a contraction that lasts for 3 minutes, followed by a period of relaxation -contractions that last for 60 seconds with a 1 minutes rest in between -contractions that last for 30 seconds with a 2 1/2 minute rest in between -contractions that last for 45 seconds, with a 3 minute rest in between

-contraction of longer and stronger duration in the fundus to minimal or nonexistent toward the cervix

Which of the following describes the normal uterine contraction pattern? Answer -contraction of equal strength but different durations throughout the uterus -contraction of longer and stronger duration in the fundus to minimal or nonexistent toward the cervix -contraction of longer and stronger duration in the cervix to minimal or nonexistent toward the fundus -contraction of equal strength and equal duration at the fundus and cervix

DHA

Which of the following fatty acids is important in the development of an infant's central nervous system and vision? - palmitic acid - oleic acid - stearic acid - Docosahexaenoi acid (DHA)

-repeated late decelerations with loss of short-term fetal heart variability

Which of the following findings of fetal heart assessment is most ominous? Answer -persistent fetal tachycardia -repeated late decelerations -repeated late decelerations with loss of short-term fetal heart variability -repeated variable decelerations with "shoulders" (small accelerations right before and after the decelerations)

absence of bowel sounds

Which of the following findings would be considered abnormal in a newborn at approximately 1 1/2 hours following birth? Answer a state of deep sleep a light murmur audible on auscultation absence of bowel sounds a heart rate of 108 BPM

meat

Which of the following increases absorption of nonheme iron? Answer milk tea meat egg whites

-the area should be cleansed from front to back

Which of the following instructions should be given to a postpartal mother about perineal care? -cleansing the area should be a sterile procedure -the area should be cleansed from front to back -stitches should be soaked twice a day in a warm sitz bath for 10 minutes -an antiseptic should be used to cleanse the perineum to prevent infection

-respiratory alkalosis

Which of the following is NOT an effect of cold stress on the newborn? Answer -metabolic acidosis -pulmonary vasoconstriction -hypoglycemia -respiratory alkalosis

headaches and visual disturbances

Which of the following is a late sign of severe pre-eclampsia? Answer -ankle edema -headaches and visual disturbances -skin that bruises easily -blood pressure of 140/90

nausea

Which of the following is a presumptive sign of pregnancy? Answer -Goodell's sign -FHT with doppler -nausea -Hegar's sign

Goodell's sign

Which of the following is a probable sign of pregnancy: Answer -Goodell's sign -FHT with doppler -amenorrhea -mother reporting fetal kicks at 16 weeks

-intrauterine infection of the fetus with the virus

Which of the following is not a potential complication of maternal human papillomavirus (HPV) infection during pregnancy? Answer -increase in number and size of warts -excessive bleeding during delivery if laceration extends to affected region -intrauterine infection of the fetus with the virus -intrapartum laryngeal infection of neonate

montgomerys tubercules

Which of the following is not a probable sign of pregnancy? Answer Braxton-Hick's contractions Montgomery's tubercles positive pregnancy test Goodell's sign

They prevent brain damage from occurring during vaginal birth

Which of the following is not one of the purposes/functions of the sutures and fontanels on the fetal skull - they prevent brain damage during vaginal birth - they allow for flexibility and mobility of the cranial bones - they enable the fetal skull to pass through the birth canal - they aid as landmarks to assess positioning of the fetus in utero

HSV

Which of the following is not thought to be a risk for cervical cancer? Answer human papilloma virus herpes simplex virus smoking multiple sex partners

eat a dry carb such as crackers before getting out of bed

Which of the following is the best recommendation for a woman with morning sickness? -nothing will really alleviate it; counsel her to just accept it -eat a dry carbohydrate, such as crackers, before getting out of bed -drink large quantities of fluids with meals, especially water -eat three meals a day and avoid eating between meals

-occipitomental

Which of the following is the largest diameter of the fetal head? Answer biparietal occipitofrontal suboccipitobregmatic occipitomental

occipitomental

Which of the following is the largest diameter of the fetal head? - biparietal -occipitofrontal -occipitomental - suboccipitobregmatic

vitamin D deficiency can cause pregnancy complications

Which of the following is true about nutrition? Answer -by the end of the eighth month of pregnancy, the fetus has a fully functioning brain, so the mother's nutrition is not as important as it was earlier in the pregnancy -it is best for a pregnant woman to eat well but avoid putting on any extra fat -vitamin D deficiency can cause pregnancy complications -low protein, high carbohydrate diet is recommended to avoid pre-eclampsia

the bleeding is usually painless

Which of the following is true about placenta previa? Answer -the bleeding is usually painless -the bleeding is usually very painful -the bleeding is generally dark red -the bleeding will usually stop as the baby descends into the pelvis

-it is most noticeable in women with darker skin and hair

Which of the following is true about the condition of chloasma? Answer -it is the same thing as linea negra -it occurs only during pregnancy -it is most noticeable in women with darker skin and hair -it is considered a precursor to melanoma if it occurs on the chest

The cord is wrapped around the baby's neck approximately 1 out of every 3 births

Which of the following is true about the umbilical cord? -The cord contains 2 veins and 1 artery -Cord entanglement is the leading cause of miscarriage before 12 weeks gestation -The cord is wrapped around the baby's neck approximately 1 out of every 3 births -Knots in the cord are common, occurring around 25% of all births

Glucose, amino acids, vitamins and minerals, fatty acids, IgG antibodies, viruses

Which of the following may be transported through the placenta - Glucose, amino acids, vitamins and minerals, fatty acids, IgG antibodies, viruses -Glucose, large bacteria, vitamins and minerals, fatty acids, IgG antibodies, viruses Glucose, IgM antibodies, vitamins and minerals, fatty acids, IgG antibodies, viruses Glucose, HIV, vitamins and minerals, fatty acids, IgG antibodies, viruses

4 oz of beef, 1/2 cup lima beans, a glass of milk, and 3/4 cup strawberries

Which of the following menus would provide the highest amounts of protein, iron, and vitamin C: Answer -4 oz of beef, 1/2 cup lima beans, a glass of milk, and 3/4 cup strawberries -3 oz chicken, 1/2 cup corn, a lettuce salad, and a small banana -1 cup macaroni, 3/4 cup peas, a glass of juice, and a medium pear -1 scrambled egg, 1 cup potatoes, half a glass of buttermilk, and a large nectarine

the CNS

Which of the following organs or organ systems is highly susceptible to teratogenic influences over the longest period of time - the CNS - the cardiovascular system - the ears - the eyes

herpes

Which of the following sexually transmitted diseases is viral rather than bacterial? Answer herpes gonorrhea syphilis chlamydia

jitteriness

Which of the following signs indicate hypoglycemia in the newborn? -blotchy skin -hypertonia -jitteriness -soft, weak cry

hypertrophy

Which of the following terms is used to describe an increase in the size of existing cells? Answer -hypertrophy -hyperplasia -hypermeiosis -hypermetosis

white or clear mucous discharge

Which of the following types of vaginal secretions are normal throughout pregnancy? Answer white or clear mucous discharge foamy, yellow discharge with foul odor clumpy, thick, white discharge with itchiness red tinged discharge

-a breastfeeding woman more than a week postpartum

Which of the following women is most likely to suffer from mastitis? Answer -a breastfeeding woman in the first 7 days postpartum -a breastfeeding woman more than a week postpartum -a non-breastfeeding woman in the first 7 days postpartum -a non-breastfeeding woman more than a week postpartum

-try massaging the baby's tummy in a sweeping circular motion

Which of the following would be an appropriate non-allopathic treatment for a newborn with colic? Answer -try a half dose of acetaminophen -try a few drops of Echinacea since colic often means the baby has an infection -try massaging the baby's tummy in a sweeping circular motion -try a few drops of whiskey to calm the gastric nerves

heart rate below 110

Which of the following would be considered bradycardia in a fetus? Answer heart rate above 120 heart rate below 180 heart rate below 110 heart rate above 160

ask her to get an ultrasound

Which of the following would be part of the midwife's work up to definitively rule out twins in a woman who is measuring large for dates: Answer -ask her if she is feeling more than normal movement -ask her if she has had large babies in the past -ask her if she has a family history of twins -ask her to get an ultrasound

urinary frequency

Which of these signs would NOT be indicative of a ruptured tubal pregnancy? Answer marked lower abdominal pain vaginal bleeding urinary frequency shoulder pain

-cord does not retract when pressing above the pubic bone

Which one of the following is a sign of placental separation? Answer -contractions every 2 minutes -cord is still pulsing -cord does not retract when pressing above the pubic bone -cord retracts when pressing above the pubic bone

calcium

Which one of the following is essential in the intrauterine development of the bones, brain and nervous system, teeth and gums, as well as proper blood circulation? Answer calcium vitamin A iron magnesium

IUD

Which one of the following is not a type of barrier birth control: Answer diaphragm IUD condom cervical cap

protein

Which one of the following is the most important nutrient of the pregnant and lactating woman by helping to build strong muscles, adequate blood volume, and healthy skin? vitamin C protein magnesium folic acid

-high levels of human chorionic gonadotropin (hCG

Which physiologic change of pregnancy probably causes first trimester nausea and vomiting? Answer -high levels of human chorionic gonadotropin (hCG) -increased gastric motility -reflux of acidic secretions into the lower esophagus -altered stomach position

the isthmus

Which portion of the fallopian tube is the site of sterilization? Answer the infundibulum the ampulla the interstitial segment the isthmus

hands and knees

Which position is recommended for a woman with severe back labor? Answer knee-chest left lateral dorsal hands and knees

uterine tenderness, fever, and foul smelling lochia

Which postpartum assessment finding would suggest endometritis in a woman whose amniotic membranes ruptured 48 hours before the onset of labor? Answer -back pain, fever, and dysuria -fever and calf tenderness -unilateral, purplish perineal discoloration and vulvar pain -uterine tenderness, fever and foul smelling lochia

-action of Doderlein's bacillus

Which principle factors cause the vaginal pH to be acidic? Answer -cervical mucus changes -secretions from the Skene's gland -action of Doderlein's bacillus -secretions from Bartholin's gland

amnioscopy

Which procedure allows for examination of the amniotic fluid through the intact membranes? Answer amniocentesis amnioscopy amniotomy fetoscopy

what is your normal daily food intake

Which question should the midwife ask a woman during the nutritional assessment part of her initial prenatal exam? Answer "Which foods seem to cause you the most discomfort?" "Which foods do you eat from the four basic food groups?" "What is your normal daily food intake?" "How much milk do you usually drink?"

she should consume alcohol only in small amounts

Which statement about a woman's nutrition during breastfeeding is not correct? Answer -she should consume alcohol only in small amounts -she should drink 6-8 glasses of fluids daily -she should have 4-6 servings of milk products daily -she should increase her caloric intake by approximately 500 calories a day

-it may cause hypotensive episodes

Which statement about early ambulation following birth is true? Answer -it may cause hypotensive episodes -it facilitates uterine involution -it requires no assistance -it should not be attempted for at least 4 hours after delivery

-the primary oocyte contains the haploid number of chromosomes

Which statement about oogenesis is incorrect: Answer -oogonia differentiate into primary oocytes during early fetal life -the primary oocyte contains the haploid number of chromosomes -the first maturation division results in a secondary oocyte and first polar body final maturation of the secondary oocyte is completed at fertilization

they look like bruising on the lower back

Which statement is true about Mongolian spots: Answer -they look like bruising on the lower back -they are diffuse edema of the scalp resulting from compression of the local blood vessels -they appear as strawberry marks around the spine -they require a physician consult since they can indicate an infection

-syphilis can pass through the placenta from mother to baby

Which statement is true regarding complications in pregnancy or labor and delivery? Answer -poor nutrition in the mother may lead to a small baby but an easier labor and delivery -VBAC births are likely to suffer a uterine rupture -syphilis can pass through the placenta from mother to baby -breech births are more difficult because the largest fetal part comes out first

DNA

Which structure carries the chromosomes' genetic information? Answer ribonucleic acid deoxyribonucleic acid centrioles amino acid

corpus

Which term refers to the body of the uterus? Answer cervix fundus isthmus corpus

labia majora

Which term refers to the externally visible structures of the female reproductive system, which are formed by two folds of fatty tissue covered with skin, extending from the mons pubis anteriorly to merge with the muscles of the perineum, and protect the inner genitalia? Answer mons pubis vestibule outer cervical os labia majora

rugae

Which term refers to the thick folds of membranous stratified epithelium on the internal vaginal wall, capable of stretching during the birth process to accommodate delivery of the baby? Answer external os fornices dartos muliebris rugae

round

Which type of ligament tends to hypertrophy during pregnancy, helping to stabilize the uterus? Answer round cardinal uterosacral broad

funic souffle

While listening to fetal heart tones, the midwife counts 130 and detects a synchronous whistling sound. These findings indicate: Answer uterine soufflé funic soufflé fetal bradycardia fetal tachycardia

contraindication

a reason something is not advisable or should not be done

erythrocyte

a red blood cell is called __________ - leukocyte - erythrocyte -polycythemia - capillary

late decelerations

a response to transient decreases in oxygen tension the nadir of the bradycardia is late relative to the peak of the contraction if recurrent it is associated with uteroplacental insufficiency

cervical cap

a rubber cap that fits over the cervix, used with spermicide to prevent conception.

petechiae

a small red or purple spot caused by bleeding into the skin.

hematoma

a solid swelling of clotted blood within the tissues.

antigen

a substance that induces and immune response is called - antigen - antibody - b- cell - lymphocyte

tubal ligation

a surgical sterilization procedure in which the fallopian tubes are sealed or cut to prevent sperm from reaching a mature ovum

28 days

a typical menstrual cycle covers a period of about - 14 days -21 days -28 days -35 days

gestational diabetes

a vegan client may be at higher risk for all of the following except - an inadequate intake of protein for meeting the demands of pregnancy - inadequate b12 production - gestational diabetes

decrease fluids

a woman c/o hemorhhoids. the midwife might recommend all of the following except - decrease fluids - side lying positions - sitz baths -anesthetic ointment

salpingitis

a woman has become infertile due to chronic inflammation of her uterine tubes from frequent infection with STDS this tubal inflammation is called? - endometritis - perimetritis - salpingitis - proctitis

vaginal prolapse

a woman is 6 months pp and c/o incontinence. all of the following should be ruled out except - stress incontinence - urge incontinence -rectocele, urethrocele, cystocele, etc. - vaginal prolapse

apply and ice pack to her perineum

a woman rpts perineal pain after birth. a good recommendation would be for her to: -increase her fluids -massage her perineum - nurse more often - apply an ice pack to her perineum

primipara

a woman who has given birth once

nullipara

a woman who has never given birth

she may have a hematoma in her vagina and possibly an infection

a woman who is 24 hrs pp complains of fever and chills and rpts pain and a feeling of pressure in her groin. the midwife would suspect: - she has been sitting too long and is getting positional pain witht he minor discomforts of lactation - she may have a hematoma in her vagina and possibly an infection -she may have a UTI - she may be constipated which is normal after delivery

incidence of UTI

abdominal muscles specifically the tone of the muscles can have affect on pregnancy . Which is not an affect of the the abdominal muscles in pregnancy? -incidence of UTI - incidence of backache - incidence of malpresentation - incidence of pendulous abdomen

diaphragmatic hernia

abnormal displacement of organs through the muscle separating the chest and abdomen

hypothermia

abnormally low body temperature

shoulder

acromial refers to which body part - cheek -neck -groin -shoulder

naegeles rule

add 7 days to LMP, subtract 3 months, add 1 year

stem cells

all differentiated cells originate from - sex cells - somatic cells -stem cells

decrease in distensibility

all of the following are effects of pregnancy on the vagina except - decrease in distensibility - darker coloration due to increased blood flow - increased sensitivity - increased discharge

safe handling of contaminated laundry includes wearing a mask, goggles, gloves, or water-proof gown, foot covering, hair covering and using plastic bags to keep wetness off clothing

all of the following are true about OSHA guidelines postpartum except - hazardous waste consists of sharps, and disposable pads contaminated with blood or bodily fluids - safe handling of contaminated laundry includes wearing a mask, goggles, gloves, or water-proof gown, foot covering, hair covering and using plastic bags to keep wetness off clothing -cleaning supplies such as bleach, ammonia, etc. require safe handling and teaching about what to do in case of skin contact or mucousal contact - human tissue such as placenta and bloody chux need to be disposed through biomedical waste

morula

an 8 celled embryo would be considered a: - blastocyst - zygote - morula - blastomere

Hypocalcemia

an abnormally low level of calcium in the blood

retroverted uterus

an anterior cervix in a non pregnant women indicates: -retroverted uterus -antiverted uterus -anterior uterus -bicornate uterus

penny royal

an example of an herb that is contraindicated for pregnancy is -alfalfa -pennyroyal -chamomile -peppermint

amnionitis

an inflammation of the amnion

Depo Provera Shot

an injectable hormonal contraceptive, it lasts for 3 months. 99% effective at preventing pregnancy only if taken exactly 3 months apart each time

immediately after birth once the baby has established respiration

babies should be allowed to breastfeed - only after newborn exam is done to ensure no digestive tract defects - 30 min after birth once the placenta is delivered and maternal bleeding is stable -immediately after birth once baby has established respiration - after the first hour after birth has passed and the mother is cleaned up, fed, and rested a it

both identical chromosomes and identical DNA information

because of mitosis and cytoplasmic division, the resulting daughter cells contain - identical chromosomes - identical DNA info - both identical chromosomes and identical DNA info - neither identical chromosomes nor identical DNA

vasectomy

bilateral surgical removal of a part of the vas deferens

IUD (intrauterine device)

birth control method, a small device inserted inside of the uterus

chemical birth control methods

birth control, transdermal patch, vaginal ring, norplant, injectable contraceptives

breech presentation

birth position in which the buttocks, feet, or knees emerge first

cystocele

bladder descent into the vagina is termed - diverticulum -hydrocele -cystocele -rectocele

from right atrium to right ventricle to lungs to left atrium to left ventricle

blood flows through the heart in which of the following general pathways - from right atrium to right ventricle to lungs to left atrium to left ventricle -from right atrium to left atrium to lungs to left ventricle to right ventricle - from left atrium to right atrium to lungs to right ventricle to left ventricle - from left ventricle to lungs to right atrium to right ventricle

placental souffle

blowing sound when auscultating over the placental location. Circulation through dilated uterine arteries, synchronous with maternal heart rate

3rd day

breastfed babies will still have some meconium in the diaper through the: - 1st day -3rd day - 5th day - 7th day

Tachypnea in newborn

breathing faster than 60 breaths/min

carbon dioxide decreases

breathing rate is most likely to increase if the blood concentration of - carbon dioxide decreases - carbon dioxide increases - hydrogen ions decrease - oxygen increases

cheek

buccal refers to which body part - nose - hips - buttocks - cheek

fats- food sources

butter, vegetable oils, meat fats, milk products, nut/seeds

VAGUS

cardiac muscle, swallowing, and gastrointestinal glands are all affected by which cranial nerve? - abducens - vagus - trigeminal - hypoglossal

etiology

cause of disease

prophase

centrosomes migrate to opposite poles of the cell during which phase of mitosis? -prophase -metaphase - anaphase -telephase

5

cephalization begins during week - 3 - 4 - 5 -6

menopause

cessation of menstruation

positive signs of pregnancy

changes that leave no doubt that the changes are due to a pregnancy in progress - positive - probable - presumptive

risk factors

characteristics or behaviors that increase the likelihood of developing a medical disorder or disease

megaloblastic anemia

chloasma can be a sign of what type of anemia? - megaloblastic - microcytic - normocytic

glucose

chromium assists in ________ uptake - calcium - sodium - potassium - glucose

vitamin c-food sources

citrus fruits, acerola cherries, alfalfa sprouts, cantaloupe, strawberries, broccoli, tomatoes, green peppers

test her for UTI, do a fern test for ROM, and a urinalysis for proteinuria, glucosuria and ketonuria

client calls with ctx 7-12 min apart and thinks she is in labor. Midwife should do all of the following except - determine weeks gest, ask about bloody show, have her describe the location of pain during ctx - have her try things like eating, resting, walking and bathing to see if these activities increase or decrease the ctx - try to elicit any evidence of the status of her membranes and her comfort during urination - test her for uti, do a fern test for ROM, and a urinalysis for proteinuria, glucosuria and ketonuria

Vitamin E- Food sources

cold pressed oils, eggs, wheat germ, organ meats, sweet potatoes, leafy vegetables

mongolian spot

common variation of hyperpigmentation in Black, Asian, American Indian, and Hispanic newborns

android pelvis

commonly known as the male pelvis - in 32.5% of white women - in 15.7% of non white women - increases risk for posterior baby - heart shaped inlet - sacrum is anteriorly inclined and flat - ischial spines are prominent and encroaching - pubic arch is narrow with an angle of much less than 90 degrees

Iatrogenic complications

complications inadvertently caused by practitioners or by medical treatments or procedures

pica

compulsive eating of nonnutritive substances such as clay or ice

foramen ovale

connects the two atria in the fetal heart

ductus venosus

connects the umbilical vein to the inferior vena cava, bypassing the liver

increased water, exercise, and natural sources of iron

constipation can be treated with - calcium, warm moist heat, exercise - accupressure wrist bands - vitamin E, support stockings, and elevated legs - increased water, exercise, and natural sources of iron

prolonged stomach emptying time and decreased intestinal motility

constipation during pregnancy is usually caused by - prolonged stomach emptying time and decreased intestinal motility - increased peristalsis and flatulence - reflux of acidic gastric contents

condom

contraceptive device consisting of a thin rubber or latex sheath worn over the penis during intercourse

folic acid- food sources

dark leafy vegetables, organ meats, nutritional yeast, root vegetables, milk, whole grains

respiratory alkalosis

decreased levels of CO2 during pregnancy may result in - metabolic acidosis - metabolic alkalosis - respiratory acidosis - respiratory alkalosis

shoulder dystocia

delayed or difficult birth of the fetal shoulders after the head is born

lifting the head higher than 45 degrees

developmental milestones the newborn should reach in the first 6 weeks include all of the following except - attentiveness to a face - lifting head higher than 45 degrees - spontaneous vocalizations

attachment disorder

developmentally inappropriate behaviors in which a child is unable or unwilling to form normal attachment relationships with caregiving adults

caput succedaneum

diffuse edema of the fetal scalp that crosses the suture lines. reabsorbes within 1 to 3 days

plethora

excess

polycythemia

excess of red blood cells

hyperbilirubinemia

excessive level of bilirubin in the blood

extension

fetal chin tips upward

marked bradycardia

fhr <100 for 10 minutes or more

lanugo

fine, soft hair, especially that which covers the body and limbs of a human fetus or newborn.

complete breech presentation

flexion at the thighs and knees, feet and buttocks or feet present

dark green leafy vegetables

folate can easily be obtained from which of the following groups of foods - dark green leafy vegetables - citrus fruits and berries - deep yellow and orange vegetables - dairy products and eggs

effects of progesterone and decreased abdominal muscle

following birth a womans bowels tend to be sluggish due to: -effects of progesterone and decreased abdominal muscle - psychological fear - effects of estrogen -presence of hemorrhoids

FPAL

full term, premature, abortion, living

proper bonding always prevents future child abuse

maternal and child bonding are key to human life for all of the following reasons except - proper boding always prevents future child abuse - proper bonding increases likelihood of successful breastfeeding - proper bonding helps with the process of grieving if an infant dies -proper boding helps assure the infants survival

protein-food sources

meats, fish, poultry, soy, eggs, milk products, beans

b6 vitamin- food sources

meats, whole grains, organ meats, molasses, wheat germ, legumes, green leafy vegetables

inflammation of the membranes around the brain

meningitis refers to: - inflammation of the brain -inflammation of the membranes around the brain - inflammation of the spinal cord - an inflammation unique to males

chin

mental refers to which body part - brain - chin - neck - shoulder

dehydration

michelle is a G1, 10 weeks pregnant . SHe reports mild cramps 2 days prior, but no bleeding, her urine shows the following: SG>1.030, pH 6.5, +1 Protein, - glucose, +1ketones, - leukocytes, - nitrites, - blood. You suspect what? - diabetes - UTI - Dehydration - pre-eclampsia

sinciput

military or brow presentation

50%

moderate colonization levels of GBS in the mother increase the risk of newborn colonization to about - 10% -25% -50% -75%

anthropoid pelvis

most common in non white women 40.5%, 23.5 % of white women - has the longest sacrum - favors a posterior baby - oval inlet - sacrum is posteriorly inclined - ischial spines are prominent but not encroaching -pubic arch is somewhat narrow

gynecoid pelvis

most favorable pelvis for successful labor -41-42% of women - has a rounded inlet - sacrum is parallel with the symphysis pubis - ischial spines are blunt and neither prominent nor encroaching -wide pubic arch greater than 90 degrees

endorphins

natural morphine like substances secreted by neurons in response to pain are called - adrenaline - endorphins - relaxin - energy

brachial plexus

network of interlacing nerves found in the upper arm area

sterile

no living microbes

eutocia

normal labor

4-8 ounces per week

normal weight gain in a newborn is - 6-12 oz per week - 4-8 oz per week - 2-3 oz per week - 10-14 oz per week

b vitamins- food sources

nutritional yeast, whole grains, brown rice, molasses, organ meats, meat, fish, poultry, egg yolks, legumes, nuts

variable decelerations

occurs secondary to an interruption in umbilical blood flow (cord compression) - common in the second stage of labor - can be episodic or periodic - the most common variant FHR pattern (45%-75%) of all labors

scanty, less than normal menstrual flow

oligomenorrhea refers to - painful menstruation -cessation of regular menstrual cycles -a missed period - scanty, less than normal menstrual flow

footling breech

one or both feet are present first at the cervix

adequate vascularization of placental bed by maternal arteries and fetal blood circulation to placenta

optimal blood exchange between placenta and fetus is dependent on what factors - the size and thickness of the placenta - fetal cardiac and circulatory development - adequate production of the hormone hpl -adequate vascularization of placental bed by maternal arteries and fetal blood circulation to placenta

bilirubin

orange-yellow pigment in bile; formed by the breakdown of hemoglobin when red blood cells are destroyed

eye

orbital refers to which body part - nose -ears -eyes - throat

testis

orchiditis is the term for inflammation of a/an - ovary -penis -testis -uterus

b12- food sources

organ meats, fish, pork, eggs, cheese, milk products, fermented soy products

ears

otic refers to what body part - eyes - lips - back - ears

chorion

outermost layer of the fetal membrane

Hyper-reflexia

over active response to reflexes

chloasma

pigmentary skin discoloration usually occurring in yellowish brown patches or spots

placenta increta

placenta penetrates into myometrium

chorionic villi invade through the entire uterine wall to the serosa layer

placenta percreta occurs when: -chorionic villi invade through the entire uterine wall to the serosa layer - chorionic villi extend further than the myometrium and penetrate the uterine wall

an increase in red blood cells

polycythemia is: - an increase in red blood cells - an increase in white blood cells - a side effect of newborn hemorrhagic disease - a glucose disorder

third to 6th day following delivery

postpartum blues may be manifested by mood swings, anger, difficults sleeping, and most commonly occurs when - 1-2 days pp - on the day of birth - 3-6 day pp - 6 weeks pp

3 P's of labor

power, (uterus) passenger, (baby) passage (pelvis)

proteinuria

presence of protein in urine

ovaries and placenta

progesterone is secreted in which location - anterior pituitary - ovaries/placenta - hypothalamus/ placenta - adrenals/ ovaries

femoral pulse

pulse of the femoral artery felt in the groin

zinc- food sources

pumpkin/sunflower seeds, seafood, organ meats, mushrooms, nutritional yeast, soybeans, oysters, herring, eggs, wheat germ, meat

eye prophylaxis is meant to prevent blindness due to syphilis

the following are true about newborn physical care except - eye prophylaxis is meant to prevent blindness due to syphilis - umbi cord care is based on promoting drying of the cord and cleanliness at its base - neonatal urination should occur in the first 24 hours of life - the establishment of breastfeeding in the first hour of life is the main priority for a healthy neonate

a cbc with differential is primarily used to detect type and clotting

the following are true about pp lab testing except - a cbc with differential is primarily used to detect type and clotting - hct and HgB are used to detect anemia - vaginal secretions are tested for viruses /bacteria depending on the symptoms - clean catch urine involves obtaining a midstream sample

generally, maternal ingestion of over the counter drugs and prescribed drugs does not increase fetal risk

the following are true about risks to the neonate except: - perinatal deaths are usually associated with a number of adverse conditions combined -generally, maternal ingestion of over the counter drugs and prescribed drugs does not increase fetal risk - there is increased risk to a neonate who follows a pregnancy which ended in fetal mortality or morbidity - generally, any serious maternal illness of condition contributes to fetal risk

a nosocomial infection is caused by the birth environment or the care provider

the following is true about pp complications leading to death - puerperal morbidity refers to pp complications leading to death - a nosocomial infection is caused by the birth environment or the care provider - endometritis is caused by uterine lining outside of the uterus - maternal mortality rate refers to the deaths attributed to complication pf pregnancy per 1000 women

small for GA infants are more prone to birth trauma

the following size/age conditions affect neonatal risk as follows except - preterm infants are prone to respiratory problems - post term infants are prone to asphyxia and meconium aspiration - LGA infants are prone to hypoglycemia - SGA infants are more prone to birth trauma

fetal presentation

the foremost part of the fetus that enters the pelvic inlet

placentome

the functional unit of the placenta is: - the placentome - chorionic villi - intervillus space - cotelydon

decidua basalis

the layer forming the maternal side of the placenta is known as: - decidua basalis - decidua capsularis - chorion frondosum - decidua parietalis

the functional endometrium

the lining of the uterus that thickens in preparation for a possible fertilized ovum, and is shed monthly during menstruation is called - the perimetrium - the functional endometrium - the basilar endometrium - the myometrium

head circumference

the measurement around the greatest circumference of the head of an infant

active labor

the midwife should e present and attending the woman in labor no later than when - early labor - active labor - transition -pushing

all of the above

the midwife should try to stop pp bleeding by which of the following - rub up a contraction - give a uterotonic - empty the uterus - all of the above

is flexing his arms and legs

the muscle tone of a baby is considered good if during the APGAR scoring her: - cries faintly - grimaces - sneezes - is flexing his arms and legs

anteverted and anteflexed

the normal tilt and curvature of the uterus within the pelvic cavity is - retroverted and anteflexed -anteverted and retroflexed - anteverted and anteflexed - retroverted and retroflexed

progesterone

the placenta produces which hormone to maintain pregnancy - estrogen - LH - progesterone - testosterone

hPL

the placental hormone used most often to monitor placental function is - hPL - progesterone - hCG - estrogen

lochia

the postpartum vaginal discharge that typically continues for 4-6 weeks after childbirth

inadequate maternal uteroplacental blood flow

the primary cause of placental insufficiency is: - structural anomalies of the chorionic villi - poor fetoplacental blood flow - inadequate maternal uteroplacental blood flow - abnormal number of umbilical cord vessels

transcription

the process of copying DNA info into the structure of messenger RNA is called - translation - deamination - transcription - respiration

differentiation

the process of specialization is - mitosis - meiosis - differentiation - apoptosis

1:3

the proportion of hemoglobin to hematocrit is stable at a ratio of: -1:3 -3:1 -2:1 -1:2

meiosis 1

the reduction process of chromosome number from diploid to haploid occurs during which process of cell division - mitosis -meiosis 1 - meiosis 2 - cytokensis

oligohydramnios

too little amniotic fluid

polyhydramnios

too much amniotic fluid

gravidity

total number of pregnancies

sepsis

toxic inflammatory condition arising from the spread of microbes, especially bacteria or their toxins, from a focus of infection

massage

treatment for thrombophlebitis includes all of the following except: - bed rest -hot packs -massage - elastic stockings

true

true or false an erection is triggered by a parasympathetic nerve response

false

true or false there is a decreased sensitivity to touch during pregnancy

nicotine and carbon monoxide

two compounds found in cigarettes are especially harmful in pregnancy; these two toxins account for most smoking related complications in pregnancy - asafetida and camphor -nicotine and bromine -nicotine and carbon monoxide -carbon monoxide and chlorine

telophase

two separate nuclei form during which phase - prophase -metaphase - anaphase -telophase

place newborn skin to skin contact

under normal circumstances, which of the following is the midwifes first step following birth of the baby - place newborn skin to skin contact - ensure an airway - assign apgar scores - help initiate breastfeeding

megaloblastic

which type of anemia would be treated with the following recommendation? fresh greens, nutritional yeast, b-complex tablets, spirulina, 400mcg folic acid - megaloblastic - microcytic - normocytic

ventouse

vacuum extraction

permanent birth control methods

vasectomy and tubal ligation

unsaturated fats- food sources

vegetable oils, sunflower seeds

telomeres

what are located on the tips of chromosomes limiting the number of cycles during the life of a cell - centromeres - centrioles - telomeres - sarcomeres

decreased fetal gas exchange resulting in hypoxia

what are the potential consequences of uterine tachysystole? - decreased fetal gas exchange resulting in hypoxia - increased fetal gas exchange resulting in fetal wellbeing - stable oxygenated blood flow to the uterus

reduces decelerations from head compression

what is a benefit of keeping the membranes intact in the first stage of labor - slows labor down - reduces the risks of a prolonged second stage - reduces the length of 1st stage labor - reduces decelerations from head compression

Subjective, objective, assessment, plan

what is meant by SOAP charting

chromosomal abnormalities

what is the most common cause for spontaneous miscarriage - viral infection -malnutrition -advanced maternal age -chromosomal abnormalities

it ligates the uterine blood vessels

what is the result of a firmly contracted uterus during the fourth stage of labor - it interferes with breast feedings - it causes bladder dysfunction - it increases the amount of lochia - it ligates the uterine blood vessels

small intestine

what is the site of most chemical digestion - stomach -small intestines - large intestines - liver

microcytic

what type of anemia is caused by: iron deficiency, thalassemia, or sideroblastic? - microcytic - megaloblastic - normocytic

Normocytic

what type of anemia is characterized by: increased MCV, large amounts of RBC, acute blood loss, hypothyroidism, or liver disease - megaloblastic - microcytic - normocytic

IM vitamin K is effective in preventing hemorrhagic disease of the newborn

when discussing vitmain k for the newborn with parents, which of the following should the midwife say - there is no risk associated with vitamin k injections - both oral and IM require a single dose - IM vitamin K is effective in preventing hemorrhagic disease of the newborn - vitamin k contains blood products so Jehovah's witnesses may want to refrain

cystitis is more likely to occur in an alkaline environment

when testing urine you check the ph. Which of the following is an explanation for why this info is useful - alkaline urine is usually seen with a diet low in carbs - acidic urine is strongly associated with diabetes - cystitis is more likely to occur in alkaline environments - cystitis is more likely to occur in acidic environments

cephalopelvic disproportion

when the fetal head is larger than the maternal pelvic opening

mouth and small intestines

where does most carbohydrate digestion occur? - mouth and stomach - stomach and small intestines - mouth and small intestines - mouth and large intestine

AB Blood type

which blood type is the universal recipient? - A - B -AB - O

syphilis

which disease is transmitted by a spirochete treponema pallidum, progresses through four stages and affect CNS of the fetus? - gonorrhea -syphilis -HIV/AIDS - HPV

ductus arteriosus

which fetal bypass structure must constrict in order for blood to flow directly to the lungs for oxygenation - foramen ovale -ductus venosus -ductus arteriosus - umbilical vein

estrogen

which hormone controls the proliferative phase of the menstrual cycle - progesterone -luteinizing hormone -hCG -Estrogen

exaggerated reflexes

which id not a possible result of inadequate T4 levels? - miscarriage - hypertension - exaggerated reflexes - pp hemorrhage

ovarian cyst

which is more common in women of child bearing age - urethral diverticula -periurethral cysts -ovarian cyst

wearing a tight fitting bra

which is not a comfort measure for a pp woman with sore cracked nipples -wearing a tight bra -expressing milk and rubbing it on nipples -correcting latch/position -lanolin cream rubbed on nipples

decreases glucose sensitivity

which is not a function of HcG? - Decreases glucose sensitivity - maintains the corpus luteum - stimulates production of placental progesterone - stimulates maternal thyroid gland - promotes myometrial growth

the urinary bladder walls are thinner, allowing easier penetration of bacteria

which is not a reason why UTIs are more common during pregnancy - lax walls of the bladder may lead to incompetent emptying of the bladder, leading to urinary stasis - the urinary bladder walls are thinner, allowing easier penetration of bacteria - ureters elongate and twist to accommodate a larger volume of urine - ovarian arteries and veins enlarge and put pressure on ureters

diaphragm is stiff and must retract to allow lung expansion

which is not an explanation for why the neonates first breath must be forceful - lungs are collapsed at birth - lung fluid must be forced into alveoli for reabsorption into pulmonary lymph vessels - surfactant is undispersed - diaphragm is stiff and must retract to allow lung expansion

postmaturity and placenta accreta

which is not directly tied to cigarette smoking during pregnancy? - ectopic pregnancy - IUGR, low birth weight, premature rupture of membranes - post maturity and placenta accreta - neonatal death and SIDS

in inhibits implantation

which is not true about HCG - it is secreted by the trophoblast - it inhibits implantation - it is necessary to maintain the corpus luteum - it is required for survival of the blastocyst

it has a lower affinity for oxygen than adult blood

which is not true about fetal blood - it contains larger and more abundant RBCs - it has a lower affinity for oxygen than adult blood - it contains a higher concentration of hemoglobin - it has a greater volume per kg of body weight than adult blood

they are attached to the lateral aspects of the uterus

which is not true about ovaries - they are 3.5cm long, 2cm wide, and 1 cm thick - they are an ovoid shape, about the size of a dried apricot - the consist of two primary layers; the cortex and the medulla - they are attached to the lateral aspects of the uterus

smoking during pregnancy doubles the chances that a baby will be born premature

which is true about smoking in pregnancy - it doubles the chances that a baby wil be born premature - can cause macrocephaly - all SIDS deaths have been linked to smoking - gallbladder disease is more common in women who smoke

calcium

which mineral is necessary to activate the binding sites for muscle contraction to occur? - calcium - sodium - potassium - iron

milk residue

white spots ont he infants tongue and gums that can be easily removed indicate: - strep throat -milk residue - thrush - lactose intolernace

forces extension of the fetal head to facilitate birth

which of the following is not a role of the pelvic floor during childbirth - forces extension of the fetal head to facilitate birth - creates resistance against the fetal presenting part - in the presence of strong contractions forces the fetus to rotate forward - forces rotation of the fetus allowing negotiation through the maternal pelvis

trembling or shaking

which of the following is not a serious warning sign during third stage -dizziness -rising pulse -pallor and clammy skin -trembling or shaking

testis

which of the following is not an accessory male sex organ - seminal vesicle - testis - penis - prostate gland

Confirm a breech or transverse lie

which of the following is not an indication for an ultrasound to be performed between 18 and 24 weeks - a fetal anatomy scan, to rule out gross defects - confirm a breech or transverse lie - investigate for size/dates discrepancy - investigate source or cause of bleeding

it increases the surface tension at the air alveolar interface

which of the following is not true about the production of surfactant - it is required for lung inflation and gas exchange - it increases the surface tension at the air alveolar interface - fetuses born weighing more than 1000g usually have sufficient production for survival - production usually occurs in the last few weeks of pregnancy

- it provides precursors to facilitate production of steroid hormones

which of the following is not true regarding the cortex of the ovary - it contains follicles that each contains an oocyte and follicular cells - the follicular cells protect and nourish the oocyte - it provides precursors to facilitate production of steroid hormones - the follicular cells produce estrogen and progesterone

diagonal conjugate

which of the following is not usually felt while doing pelvimetry and if it is felt it could indicate a too narrow inlet - diagonal conjugate - ischial spines - coccyx - sacral curve

retained placental fragments

which of the following is the most common cause of uterine subinvolution - poor abdominal muscle tone - retained placental fragments - myomata -hematoma

the interspinous diameter

which of the following is the smallest pelvic diameter to which the fetus has to accommodate itself - the obstetrical conjugate -the conjugate vera -the interspinous diameter - the intertuberous diameter

protein is too large to pass through the filtration membrane

which of the following is true regarding protein in the urine? - protein is too large to pass through the filtration membrane - proteinuria indicates the glomerulus is functioning properly - filtration pressure forces proteins from blood and out of the glomerulus

it is a natural process of discarding an extra haploid set of chromosomes

which of the following is true regarding the formation of polar bodies during oogenesis - it is a rare example of wasted energy and cellular material - it is a natural process of discarding an extra haploid set of chromosomes - three polar bodies are formed at the end of meiosis 2 - the first polar body contains a diploid set of chromosomes

round ligament

which of the following ligaments does not support the ovaries in the pelvic cavity - meovarium - round ligament - suspensory ligament - ovarian ligament

shivering that occurs as body temp drops

which of the following provides the best example of a homeostatic mechanism - shivering that occurs as body temp drops - increasing body temp during exercise - varying body temp during illness - decreasing body temp during sleep

it takes place during the telophase of a cell's life cycle

which of the following statements regarding DNA replication is not true - it takes place during the telophase of a cell's life cycle - it involves pulling apart of a double stranded DNA molecule - unzipped strands are held apart , exposing slightly charged nucleotides - it results in two DNA molecules, each with one old strand and one new strand of nucleotides

nausea and diarrhea

which of the following symptoms is associated with consuming a megadose of vitamin C? - Night blindness - scurvy - nausea and diarrhea - erythrocyte hemolysis

is formed from flattened ectoderm cells

which of the following would not be included in a description of intervillous space - is formed from flattened ectoderm cells - blood filled spaces containing villi formed by lacunae of trophoblast - lined on both sides by syncytiotrophoblast - are separated by septa into 10-40 cotyledons

diagonal conjugate approximately 11.5cm

which pelvic diameter is considered normal -bituberous diameter of 5cm -obstetric conjugate 9cm -interspinous diameter 8cm -diagonal conjugate 11.5cm

levator ani

which pelvic floor muscles function to provide a sphincter like action in both the vagina and anal canal? - levator ani - coccygeus - bulbospongiousus - ischocavernosus

constipation up to day three following birth

which postpartum discomfort is common - prolonged pp cramping up to 6 weeks after birth - constipation up to day three following birth - excessive perspiration due to hormonal changes that lasts three months following birth - excessive bleeding that continues to soak maxi pads for 4 weeks after birth

gravidity

which term means the total number of pregnancies a woman has had regardless of their duration or outcome? - parity - gravidity - nullipara - multigravida

parity

which term refers to the number of past pregnancies that have reached viability or have been born? - gravidity - parity - grandmultiparity - primigravida

the female urethral pathway is shorter and vaginal mucosa is more hospitable to microbes than the skin of the penis

why does cystitis occur more frequently in women than men - the female bladder is larger - the female urethral pathway is shorter and vaginal mucosa is more hospitable to microbes than the skin of the penis - the male bladder wall is thicker - men are more resistant to infections

multipara

woman who has given birth to two or more children

jaundice

yellowing of skin, tissue, and fluids caused by increased levels of bilirubin in the blood

gastroenterologist

you have been having chronic pains in your upper abdomen, and your family physician refers you to a specialist in diseases of the digestive tract called a? - cardiologist - pulmonologist - neurologist - gastroenterologist


Kaugnay na mga set ng pag-aaral

Lesson Test: Lección 3, Prueba F.

View Set

Biology Ch 15 Evolution & Chapter 16 Understanding Key Concepts

View Set

chapter 22 Immunity and lymphatics Wiley plus

View Set

Supply Chain: Chapter 7 Questions

View Set

General Psychology Test #2 (Part 1)

View Set

review questions ch 24 list the enzymes that are present and state which type of food .

View Set